MN Sherpath Quizzes

¡Supera tus tareas y exámenes ahora con Quizwiz!

The nurse is caring for a patient in the post-anesthesia care unit (PACU) immediately following cesarean delivery. During fundal assessment, the patient reports discomfort and exhaustion and asks the nurse, "Why do you keep poking at me?" Which response from the nurse is appropriate? "I'll wait until you're on the postpartum floor to assess you again." "I need to do this assessment to make sure your incision is healing correctly." "This assessment must be done every 30 minutes until you transfer out of the recovery room." "A cesarean delivery puts you at risk for excessive bleeding, so it's important to assess you frequently."

"A cesarean delivery puts you at risk for excessive bleeding, so it's important to assess you frequently." Fundal assessment must be performed frequently following cesarean delivery to assess for uterine atony, which can lead to hemorrhage. Postpartum hemorrhage and uterine atony are risks of cesarean delivery.

A 42-year-old woman, who is 16 weeks pregnant with her third child, was told by her health care provider that her blood test for birth defects was abnormal and further testing would be required. The woman asks the nurse to elaborate on what possible procedures she may be scheduled to undergo. Which response by the nurse is appropriate?

"A detailed ultrasound to look for birth defects may be recommended." "A procedure that tests the cells in the amniotic fluid for genetic disorders may be recommended." "A procedure that obtains blood from the fetal umbilical cord to test for genetic disorders may be recommended." "A procedure that obtains tissue sample by inserting a needle through your abdomen guided by an ultrasound may be recommended."

The mother of a 1-day-old newborn questions the nurse regarding an adhesive dressing on the heel. Which statement is most appropriate for the nurse to make? "Don't worry, it didn't hurt the baby at all." "I will get the provider to come and explain it all to you." "That area is where blood was drawn to test for infection." "All children are tested for metabolic disorders after delivery."

"All children are tested for metabolic disorders after delivery." Metabolic screening is performed on all newborns to test for conditions that can delay physical and psychological development. The test involves drawing blood from the heel.

The antepartum nurse is caring for a patient who is scheduled for cesarean delivery. The patient reports anxiety about the procedure because she is unsure of her exact due date and whether the fetus is mature enough for cesarean delivery. Which is the best response made by the nurse to this patient? "The exact due date can be determined with a gynecological assessment." "Cesarean delivery can be performed after spontaneous onset of labor to verify gestational age." "Amniocentesis can be performed prior to cesarean delivery to verify that delivery is safe for your infant." "Ultrasound can be performed prior to cesarean delivery to verify that cesarean delivery on the scheduled date is safe for the infant."

"Amniocentesis can be performed prior to cesarean delivery to verify that delivery is safe for your infant." Amniocentesis may be done prior to cesarean delivery to verify fetal lung maturity for patients with questionable due dates.

The postpartum nurse is talking with the patient following a cesarean delivery with a Pfannenstiel skin incision and low transverse uterine incision. The patient asks the nurse to describe the procedure, since the surgical draping prevented her from seeing the operation. Which response from the nurse is appropriate? "An incision was made on the uterus that goes up toward your navel." "The layer of skin and fat is cut open and then a uterine incision is made." "Are you sure you're comfortable hearing details about the procedure?" "An incision was made on the uterus that looks a lot like the scar on your abdomen."

"An incision was made on the uterus that looks a lot like the scar on your abdomen." Low transverse uterine incision and Pfannenstiel skin incision are similar in appearance and location.

The nurse notes a newborn who is formula-fed shows signs of hunger, despite being fed every 3-4 hours. Which question does the nurse ask the parents? "Do you offer a pacifier between feedings?" "Are you diluting ready-to-use formula for bottles?" "Are you offering a bottle regularly, every 1-2 hours?" "Have you given the newborn cow's milk as a substitute?"

"Are you diluting ready-to-use formula for bottles?" Diluting ready-to-use formula decreases the amount of nutrients the newborn receives and does not satiate the newborn.

A woman at 36 weeks' gestation and in preterm labor is being sent home. Which statements about home management would the nurse include in the patient education? Select all that apply. "Avoid eating spicy foods." "Avoid sex during this time." "Only move around your house when necessary." "When lying down, make sure your feet are raised." "Notify the health care provider if your temperature is greater than 100° F."

"Avoid sex during this time." "Only move around your house when necessary." "Notify the health care provider if your temperature is greater than 100° F."

A mother asks the neonatal nurse why her infant must be monitored so closely for hypoglycemia when her type 1 diabetes was in excellent control during the entire pregnancy. How should the nurse best respond? "A newborn's glucose level drops after birth, so we're being especially cautious with your baby because of your diabetes." "A newborn's pancreas produces an increased amount of insulin during the first day of birth, so we're checking to see whether hypoglycemia has occurred." "Babies of mothers with diabetes do not have large stores of glucose at birth, so it's difficult for them to maintain the blood glucose level within an acceptable range." "Babies of mothers with diabetes have a higher-than-average insulin level because of the excess glucose received from the mothers during pregnancy, so the glucose level may drop."

"Babies of mothers with diabetes have a higher-than-average insulin level because of the excess glucose received from the mothers during pregnancy, so the glucose level may drop." (The infant of a diabetic mother (IDM) produces a higher level of insulin in response to the increased maternal glucose level; after birth it takes several hours for the newborn to adjust to the loss of the maternal glucose. A healthy newborn's glucose level does not drop significantly after birth. A newborn's pancreas usually produces more insulin as a response to the maternal glucose level, but this response is not specific to the IDM. IDMs have the same glucose stores as other newborns; their responses to the loss of maternal glucose levels differ.)

A 38-year-old nulliparous woman bearing twins asks the nurse why she is at risk for PROM. Which statement by the nurse is appropriate? "Gaining weight progressively predisposes a woman to PROM." "Since this is your first pregnancy, you have an increased risk for PROM." "Being pregnant at an advanced maternal age could precipitate PROM." "Bearing multiple fetuses in one pregnancy can lead to overdistention of the uterus, which could result in PROM."

"Bearing multiple fetuses in one pregnancy can lead to overdistention of the uterus, which could result in PROM."

A woman at 39 weeks' gestation reports concern regarding vaginal secretions that are a mixture of blood and mucus. The nurse explains that this occurs for which reason? "Bloody show" occurs as a result of cervical changes during false labor. "Bloody show" occurs as the cervix begins to soften, dilate, and efface slightly. "Bloody show" occurs as fetal pressure causes congestion of the vaginal mucosa. "Bloody show" occurs because extra amniotic fluid that accumulates during pregnancy is being excreted

"Bloody show" occurs as the cervix begins to soften, dilate, and efface slightly. "Bloody show," a mixture of thick mucus and pink or dark brown blood, may occur as the cervix begins to soften, dilate, and efface slightly ("ripening").

A woman is admitted to the Labor and Delivery unit with premature rupture of membranes at 31 weeks' gestation and is experiencing symptoms of preterm labor. The nurse has assessed the woman for signs of infection and is administering corticosteroids following an order from the health care provider. Which statement should the nurse make to explain why corticosteroids have been ordered? "Corticosteroids are administered to help delay labor." "Corticosteroids are administered to help "quiet" uterine activity." "Corticosteroids are administered to help fight infection in the mother." "Corticosteroids are administered to help speed your baby's lung maturity."

"Corticosteroids are administered to help speed your baby's lung maturity."

A nurse is assessing a newborn. The parent expresses concern about cracking and peeling of the newborn's skin. How does the nurse respond? "Cracking and peeling of newborn skin is normal." "Cracking and peeling of the skin indicates an allergy to soap." "This could indicate a serious skin disorder. Let's talk to the health care provider." "This could be a sign of dehydration. You should give the newborn small amounts of water."

"Cracking and peeling of newborn skin is normal." Cracking and peeling of the skin is a normal finding in a newborn.

The nurse is assessing a newborn and new mother. The mother tells the nurse, "I am worried about breastfeeding. I often do not have enough money to eat and want my baby to get everything he needs." What is the nurse's best response? "You will still make enough breast milk for your baby so he will get the nutrition he needs." "The baby could miss some important nutrients if you do not eat well, so you should use a formula preparation." "We can get you some supplements to ensure that you get all the nutrients you need, which will pass to your baby in your breast milk."

"Even if you miss a meal, your breast milk will have about the same amount of nutrients as if you hadn't. Let's talk about some resources that you can use to get free or low-cost food." The nurse is caring for both the mother and the newborn. Providing resources for the new mother is an important part of supporting her chosen method of newborn feeding.

The student nurse is describing formula to the clinical instructor. The nursing student demonstrates understanding of formula through which statement? "Formula is made from cow's milk, so cow's milk can be used in place of formula when more convenient." "Formula is the same as breast milk and has the same components." "Formula may be used for newborns with special dietary needs such as lactase deficiency." "Formula with iron should be reserved for newborns in vegetarian families to ensure that all nutritional needs of the newborn are met."

"Formula may be used for newborns with special dietary needs such as lactase deficiency." This statement demonstrates an understanding of formula. Formula can be made from soy for newborns who are lactase deficient and cannot process the lactose in formulas made from modified cow's milk.

The patient is discussing with the nurse her preference for a vaginal delivery for her current pregnancy. Which statement by the patient indicates the need for further teaching? "I had general anesthesia during my last c-section." "Before my last c-section, the doctor tried to induce labor." "My ultrasound showed a lot of scar tissue from my last c-section." "I had a c-section with my last pregnancy because of placenta previa."

"I had a c-section with my last pregnancy because of placenta previa." Cesarean delivery due to placenta previa is typically done using a classic uterine incision, which eliminates the option for vaginal birth after cesarean (VBAC).

The nurse is teaching a new mother on proper newborn positioning. Which statement, made by the parent, indicates that the teaching was effective? "He started coughing a lot, so I laid him down." "He likes to lie down on a pillow when he gets his bottle." "I have been keeping one hand behind his head whenever I pick him up." "The baby burps better if I lay him on his back and bounce my knee."

"I have been keeping one hand behind his head whenever I pick him up." Newborns have underdeveloped neck muscles and require continuous head support.

The mother of a newborn expresses concern regarding the appearance of the umbilical cord. Which statements, made by the mother, indicate the need for further teaching by the nurse? Select all that apply. "I should report any bleeding or oozing to the health care provider." "I will pull off the stump in a few days to keep it from getting infected." "I need to apply petroleum jelly over the area with every diaper change." "I should buy diapers one size larger to prevent irritation around the cord." "I need to clean around the cord with alcohol swabs whenever I change the diaper."

"I will pull off the stump in a few days to keep it from getting infected." The stump should not be pulled off. It will fall off without assistance within 10-14 days of delivery. "I need to apply petroleum jelly over the area with every diaper change." Petroleum jelly should be placed on the post-circumcision penis, not the umbilical cord. "I should buy diapers one size larger to prevent irritation around the cord." Parents should be taught to fold appropriately sized diapers to prevent contaminating the cord with urine. "I need to clean around the cord with alcohol swabs whenever I change the diaper." Best practice is to clean the cord with water, which can lead to a decrease in separation time and does not lead to infection.

The nurse is caring for a patient who sustained a third-degree perineal laceration during delivery. The patient reports pain with defecation and asks if anything can be done. Which response from the nurse is appropriate? "Increasing your fluid intake will make your stool much softer." "I will obtain an order for an enema to make things easier for you." "I will request an order for stool softeners to help you pass stools more comfortably." "I can give you some of the hydrocodone bitartrate and acetaminophen the health care provider ordered to help with the pain."

"I will request an order for stool softeners to help you pass stools more comfortably." Stool softeners are recommended for patients with perineal lacerations to help relieve pain with defecation and ensure the patient maintains continence.

The nurse teaching parents regarding cord care knows that teaching is effective when the parent makes which statement? "I'm glad I don't have to pull this cord stump off." "I think he will get a bubble bath when we get home." "My parents bought size 3 diapers to give more room for the cord stump." "I am going to buy petroleum jelly to apply to the area as soon as I leave."

"I'm glad I don't have to pull this cord stump off." The cord stump should not be pulled. It will fall off within 10-14 days.

Which statement made by the parents indicates that teaching was effective? "We need to clean the area three times a day." "He shouldn't have wet diapers the first day home." "If the penis has a foul odor, we could call the health care provider." "It's ok for the penis to be red and produce pus for 5 days."

"If the penis has a foul odor, we could call the health care provider." Foul odor is a sign of infection and should be reported to the health care provider.

A woman is at 38 weeks' gestation. She reports suspected signs of labor to the triage nurse. Which statement by the nurse supports the beginning of true labor? "Your contractions will decrease with activity." "The contractions will be mild and more annoying than painful." "You will feel the contractions in your front pelvic area and not in your back." "Labor contractions will occur in a consistent pattern that increases in frequency, duration, and intensity."

"Labor contractions will occur in a consistent pattern that increases in frequency, duration, and intensity." Contractions occurring in a consistent pattern of increasing frequency, duration, and intensity are a sign of true labor.

A nurse recently began working in an office that provides genetic counseling services to couples. The nurse is overwhelmed with all of the information she must know. The nurse asks a co-worker about the most important information parents want to learn. Which response by the nurse's peer is appropriate?

"Most parents want to know the possibility of a birth defect when they don't have this characteristic." Most parents frequently ask about the possibility of a birth defect. when they don't possess this characteristic defect. In response, the nurse should inform the parents that autosomal recessive disorders are carried by parents who themselves are unaffected.

The nurse is teaching new mothers about breastfeeding. Which statements made by a new mother would concern the nurse? Select all that apply. "My baby has 1-2 stools each day." "My baby's stool looks like mustard." "My baby grunts whenever he poops." "My baby's poop is the consistency of paste." "I alternate formula and breast milk when I'm working."

"My baby has 1-2 stools each day." Breastfed newborns should have at least four stools per day. This would concern the nurse. "My baby's poop is the consistency of paste." Breastfed newborns should have soft stools, but a paste-like consistency may indicate a gastrointestinal problem and should be investigated further.

Which statement does the nurse use to describe PROM or PPROM to the pregnant woman? "PROM could occur at 38 weeks." "PROM occurs in full-term infants." "PPROM does not pose a risk to the infant." "PROM is a greater risk to the mother than is PPROM."

"PROM could occur at 38 weeks."

The nurse is assessing the fundus of a patient who had a cesarean delivery. The patient reports abdominal discomfort during palpation. Which instructions should the nurse give the patient regarding abdominal pain during fundal assessment? "Turn to your side." "Empty your bladder before the fundal assessment." "Relax and breathe slowly during the fundal assessment." "Take deep breaths and hold them during the fundal assessment."

"Relax and breathe slowly during the fundal assessment." Relaxing the abdominal muscles and taking slow, deep breaths can help prevent discomfort during abdominal assessment

A patient carrying twins desires an elective cesarean delivery. Which statement indicates a need for further teaching regarding cesarean delivery? "I might need a blood transfusion after the surgery." "Having a c-section might make my hospital stay longer." "Since I am not having a vaginal birth, my babies can't be harmed during delivery." "Depending on how they do the incision, I may be able to have a vaginal delivery if I become pregnant again."

"Since I am not having a vaginal birth, my babies can't be harmed during delivery." Cesarean delivery carries risk of injury to the newborn, such as bruising, fractures, or other trauma.

The antepartum nurse is caring for a patient with a body mass index (BMI) of 30 who is scheduled for a planned cesarean delivery. The patient asks the nurse why the provider has told her that a midline vertical incision will be done for the procedure. Which explanation by the nurse is accurate? "The midline vertical incision is less likely to lead to hernia formation later in life." "The midline vertical incision will heal more quickly than a Pfannenstiel incision." "The midline vertical incision will cause less postoperative pain and allow for earlier ambulation." "The midline vertical incision is a better option for larger patients because more of the uterus is exposed."

"The midline vertical incision is a better option for larger patients because more of the uterus is exposed." For obese patients (BMI ≥ 30), a midline vertical incision is optimal because it exposes more of the uterus and allows for safer delivery.

A patient in labor is in visible distress and reports fear of pushing too hard and "tearing something." Which response from the nurse is appropriate? "Cervical ripening may help ensure the tissue does not tear." "If you feel tearing sensations, you should not push any harder." "If you feel like you might tear something, I may need to request the health care provider schedule you for cesarean delivery." "The team can't guarantee that you won't have some tearing, but your body will stretch to accommodate the baby."

"The team can't guarantee that you won't have some tearing, but your body will stretch to accommodate the baby." It is not certain that stretching will prevent tearing, but stretching is sure to occur and will greatly reduce the tearing that would occur if the patient's perineal tissues could not stretch

The antepartum nurse is caring for a patient carrying twins. While educating the patient about what to expect during labor, the patient states, "I thought all twins were delivered by C-section." Which response from the nurse is appropriate? "Cesarean delivery is usually started after labor has begun." "There is a very realistic possibility that the twins may be delivered vaginally." "Many health care providers prefer to deliver one baby vaginally and one by cesarean section." "That's correct. The latest evidence shows that cesarean delivery is the safest option for multiple births."

"There is a very realistic possibility that the twins may be delivered vaginally." Although twins are often delivered by cesarean delivery, it is possible that delivery may be vaginal. It is also important that the nurse know the plan of care and history of the patient so that he or she does not contradict a patient-centered plan that was already in place.

A patient having a scheduled cesarean delivery asks the nurse why the provider plans to use a Pfannenstiel incision (transverse skin incision). Which is the most accurate response by the nurse? "There is usually a longer recovery time with a Pfannenstiel incision." "There likely will be less post-operative pain with a transverse incision." "The transverse incision allows for more space and better access to the fetus." "The transverse incision allows for faster uterine access, thus speeding up the delivery."

"There likely will be less post-operative pain with a transverse incision." A low transverse incision causes less disruption of abdominal wall musculature, and thus is less painful than a vertical, midline incision.

After delivery with a vacuum extractor, a patient asks the nurse about the red, swollen area on the side of the infant's head. Which response from the nurse is appropriate? "This is most likely the spot where the vacuum extractor was placed." "This is where the baby's head pressed up against the pelvis during extraction." "This may be a sign of a serious complication. I will notify the health care provider." "This occurs naturally because the newborn's head is compressed during delivery."

"This is most likely the spot where the vacuum extractor was placed." The vacuum extractor forms a round, edematous spot on the infant's scalp, called "a chignon," which resolves quickly after birth.

The parents of a newborn ask the nurse why the newborn must receive Vitamin K soon after birth. Which response by the nurse is appropriate? "To prevent infection." "To provide pain relief." "To treat hypoglycemia." "To activate clotting factors."

"To activate clotting factors." Newborns are born with inadequate clotting factor. Vitamin K is necessary for the production of clotting factors by the liver and is administered to prevent bleeding problems.

The antepartum nurse is caring for a patient with a history of one cesarean birth via low transverse incision. The patient asks the nurse what her chances are of a successful vaginal birth for the current pregnancy. Which is the appropriate nursing education for this patient? "Because of the risk of cervical laceration with low transverse incisions, vaginal birth after cesarean is not recommended." "With low transverse incisions, vaginal birth after cesarean is a possibility, because of a decreased risk of incision rupture." "Because of the increased risk of infection, vaginal birth after cesarean is not indicated for patients with a history of low transverse incision." "Vaginal birth after cesarean is not a possibility for patients with a history of low vertical incision, because the uterine incision may lead to lots of scar tissue."

"With low transverse incisions, vaginal birth after cesarean is a possibility, because of a decreased risk of incision rupture." Vaginal birth after cesarean is a possibility for patients with a history of low transverse incisions due to a decreased risk of uterine rupture.

A mother who breastfeeds voices concern regarding her newborn receiving enough vitamins and nutrients. Which responses by the nurse are appropriate? Select all that apply. "You may need to give iron supplements." "Add more fiber-rich foods to your diet." "The breast milk hasn't changed since you started lactating." "Exclusively breastfed newborns should receive Vitamin K supplements." "Your breast milk has enough vitamins and minerals but is deficient in immune properties."

"You may need to give iron supplements." Breast milk is often deficient in iron, and newborns may require supplementation . "Exclusively breastfed newborns should receive Vitamin K supplements." Breast milk is often deficient in Vitamin K, and newborns may benefit from supplementation.

The mother of a newborn who has a 2-year-old at home voices concern regarding the newborn's sleep schedule. Which is the appropriate statement to include in teaching the patient? "You should set a strict schedule for your baby's naps." "During active sleep, if the baby moves, she is waking up." "During the drowsy state, the baby is trying to fall asleep." "Your baby will not be disturbed by other children during quiet sleep."

"Your baby will not be disturbed by other children during quiet sleep." During quiet sleep, the newborn is not disturbed by noise.

The nurse is caring for a 36-year-old pregnant woman at 16 weeks whose health care provider recommended genetic counseling due to her advanced maternal age. The patient states, "I just don't know what to do; what if there is something wrong?" Which response by the nurse is appropriate?

"Your concern is valid. Genetic counseling could raise issues that are hard to talk about or deal with." Genetic counseling can raise issues that are uncomfortable and can lead to family conflict. The nurse can help the parents weigh the issues surrounding seeking genetic counseling.

A patient whose labor was complicated by shoulder dystocia reports that one of her infant's arms is "floppy." The nurse examines the infant and finds flaccid muscle tone on one side. Which is the most appropriate nursing education for this patient? "This is a normal effect of shoulder dystocia and should resolve in a few days." "Your health care provider will likely recommend physical therapy to treat this injury." "This could be a symptom of a permanent neurologic condition. I will notify the health care provider immediately." "Your baby's collarbone may have fractured as a result of shoulder dystocia. I will notify the health care provider immediately."

"Your health care provider will likely recommend physical therapy to treat this injury." Flaccid muscle tone in one arm is a presentation of Erb's palsy, or nerve injury to the brachial plexus. Most cases resolve in a few weeks, but physical therapy may be started in the immediate postpartum period. The nurse is justified in providing this anticipatory guidance to the patient on the basis of the health care provider's most likely recommendation.

A nurse reviews the family history of a patient that is positive for multiple birth defects, Trisomy 13, 18, and 21 and Fragile X syndrome. The patient states that she desires to become pregnant within the next 2 months and inquires about the preconception screening process. Which actions should the nurse take?

--Ask patient to bring pictures of family members. Preconception screening includes examination of family photographs for subtle or obvious birth defects. --Tell patient that she will likely undergo blood testing. Preconception screening includes chromosome and DNA analysis. A blood sample is used to complete the analysis. --Obtain history about the family's ethnic background. Knowledge of the patient's ethnic background is important as some ethnic groups have higher incidence of some genetic disorders. --Inform patient about the importance of physical exam. Preconception screening includes a physical examination to look for obvious or subtle signs of birth defects.

During the prenatal visit, the patient is informed that her fetus has Trisomy 13. The patient asks the nurse multiple questions regarding the genetic abnormality. Which actions by the nurse are appropriate?

--The nurse should discuss options for care of the infant. When a diagnosis is established, counseling should include information regarding the care of the affected person. --The nurse should discuss the natural progression of the disorder. When a diagnosis is established, counseling should include information regarding the natural progression of the disorder. --The nurse should educate the patient about the disorder and its cause. When a diagnosis is established, counseling should include information about the disorder and its cause.

What is the estimated due date (EDD) of a pregnant woman with last menstrual period onset of May 10th? Record date formatted mm/dd.

02/17 The due date (EDD) is calculated by subtracting three months from the first day of the last menstrual period and adding seven days. This is known as Naegele's rule and is based on the woman having a normal 28-day menstrual cycle.

Match the milestone with the corresponding age group 1-2 months (gross motor) 3 months (gross motor) 4-5 months (gross motor) 4-5 months (fine motor) Will reach and grasp with palm Can lift head when held against the shoulder Bears some weight when held in the standing position Can lift head off the bed when in prone position

1-2 months (gross motor) Can lift head when held against the shoulder 3 months (gross motor) Can lift head off the bed when in prone position 4-5 months (gross motor) Bears some weight when held in the standing position 4-5 months (fine motor) Will reach and grasp with palm

The nurse administered magnesium sulfate to a woman experiencing preterm labor following a health care provider order. Which assessment would concern the nurse? Urine output of 50 mL Oxygen saturation of 95% 10 respirations per minute Bronchial sounds heard over the body of the sternum

10 respirations per minute

A new mother at the clinic asks at what age her baby will be able to interact and play games like pat-a-cake with her. What would be the correct response from the nurse? 4 months 6 months 9 months 12 months

12 months At 12 months the child is likely to engage in a pat-a-cake game with the mother.

Match the abnormal finding in an infant with the causative drug if taken during pregnancy. A) Indistinct philtrum B) Tremors and sleep disturbances C) Hypertonicity, nausea, and vomiting D) Poor response to interactions with others 1) Antidepressants 2) Alcohol 3) Opioids 4) Tobacco 5) Marijuana 6) Cocaine

2) Alcohol 5) Marijuana 3) Opioids 6) Cocaine

The expected fundal height for a pregnant woman at 28 weeks gestation is ____ cm, give or take 2 cm.

28 From 18 to 30 weeks' gestation, the fundal height in centimeters is approximately equal to the number of weeks of gestation. Therefore, a woman who is 28 weeks' gestation should have a fundal height of 28 cm, give or take 2 cm.

Match the woman to the relevant high-risk condition. PTL PROM PPROM Chorioamnionitis 1) Overdistention of the uterus 2) Incompetent cervix 3) A woman at 28 weeks who is having regular uterine contractions. 4) A woman at 33 weeks who has a temperature of 101.5 degrees F and abdominal tenderness. 5) A woman at 38 weeks whose amniotic membranes have ruptured, but she has not experienced any contractions. 6) A woman at 32 weeks whose amniotic membranes have ruptured.

3) A woman at 28 weeks who is having regular uterine contractions. 5) A woman at 38 weeks whose amniotic membranes have ruptured, but she has not experienced any contractions. 6) A woman at 32 weeks whose amniotic membranes have ruptured. 4) A woman at 33 weeks who has a temperature of 101.5 degrees F and abdominal tenderness.

A nurse assesses a newborn's respiration for 60 seconds. Which describes a normal respiratory rate for a newborn? 35 breaths per minute 25 breaths per minute 20 breaths per minute 12 breaths per minute

35 breaths per minute Thirty-five breaths per minute is within the normal respiratory rate range for a newborn.

A nurse weighs a full-term newborn delivered without complications. Which results are within the normal weight range for a full-term newborn? Select all that apply. 4100 g 2400 g 3800 g 2600 g 2000 g

3800 g A healthy full-term newborn should weigh between 2500-4000 grams. A healthy full-term newborn can weigh 3800 grams. 2600 g A healthy full-term newborn should weigh between 2500-4000 grams. A healthy full-term newborn can weigh 2600 grams.

Five minutes after birth, a newborn is pale; has irregular, slow respirations; has a heart rate of 120 beats/min; displays minimal flexion of the extremities; and has minimal reflex responses. What is this newborn's Apgar score? Record your answer using a whole number.

5 (The Apgar score is 5. According to the Apgar scoring system, the newborn receives 2 points for heart rate, 0 for color, 1 for respiratory effort, 1 for muscle tone, and 1 for reflex irritability. An Apgar score of 3 is low. Scores of 5 and 6 are higher, but the newborn may still require stimulation and oxygen.)

A weight gain of 1.49 lb (0.68 kg) per month is expected from birth until which month? 6 months 8 months 10 months 12 months

6 months An infant is expected to gain 1.49 lb (0.68 kg) per month from birth until 6 months.

The nurse is observing a mother who is interacting with her infant and notices the infant trying to imitate the sounds that the mother is making. The nurse knows that the child must be at least what age? 3-4 months A child at 3-4 months will not be imitating sounds that the mother is making. The infant will be making mostly babbling sounds to self. 4-6 months At 4-6 months the child may repeat sounds to self but at this age will not be imitating sounds. 6-8 months An infant who is 6-8 months of age will be able to imitate the sounds the mother is making and therefore the nurse knows the patient must be at least this age. 1-3 months When a child is 1-3 months old they will enjoy making sounds and will respond to voices but will not be imitating the mother's sounds.

6-8 months An infant who is 6-8 months of age will be able to imitate the sounds the mother is making and therefore the nurse knows the patient must be at least this age.

The nurse is caring for a patient whose delivery was assisted by forceps. Which assessment finding should the nurse report to the health care provider immediately? Pain with defecation Fundus is firm on palpation A hard, turgid area on the labia minora Episiotomy incision flush with the surrounding skin

A hard, turgid area on the labia minora A hard, turgid area in the perineal area is a sign of a hematoma and should be reported to the health care provider immediately because it can result in significant blood loss.

The charge nurse is making patient assignments for the next shift of nurses. The nurse anticipates that induction of labor likely will be indicated for which patient? A patient at >42 weeks' gestation A patient with lupus and who is in remission A patient with adequately controlled diabetes A patient experiencing Braxton-Hicks contractions

A patient at >42 weeks' gestation Postterm pregnancy (>42 weeks) is an indication for induction of labor. Postterm pregnancy is linked with both fetal and maternal health complications; as a result, health care providers usually do everything they can to ensure that an infant is delivered as close to the due date as possible.

The nurse should anticipate preparing for an operative delivery when caring for which patient? A patient with a history of perineal laceration A patient who rates her pain at ten out of ten A patient whose bladder is distended and who is unable to void A patient who has been pushing for three hours with minimal fetal progress

A patient who has been pushing for three hours with minimal fetal progress Maternal exhaustion is an indication for operative delivery. If the patient is exhausted and making minimal progress, there may be a need to assist with forceps or vacuum.

The nurse is caring for several patients in labor. Which patient exhibits a risk that indicates the need for an operative vaginal delivery? A patient whose previous birth was assisted with the use of forceps A patient whose fetus has a heart rate between 125 and 140 beats/min A patient who stopped responding to cues to push, stating she is exhausted A patient whose most recent contractions suggest uterine hyperstimulation

A patient who stopped responding to cues to push, stating she is exhausted Maternal exhaustion may require operative delivery if the patient is unable to push sufficiently to facilitate vaginal birth without operative interventions.

The nurse is caring for two women in early labor. Which patient is at greatest risk for perineal laceration? A primipara A patient with prolonged labor A patient experiencing a vaginal delivery after cesarean delivery A patient whose delivery will be assisted with a vacuum extractor

A patient whose delivery will be assisted with a vacuum extractor Patients who experience operative delivery, which is delivery assisted by forceps or vacuum extractor, are at increased risk for perineal laceration because of the introduction of a mechanical instrument into the vagina.

The nurse is caring for a patient in the second stage of labor. Which patient condition is most likely to result in the need for an episiotomy? A patient with a history of perineal laceration A patient receiving oxytocin for induction of labor A patient whose fetus is experiencing shoulder dystocia A patient who had an episiotomy during a previous delivery

A patient whose fetus is experiencing shoulder dystocia Shoulder dystocia is an indication for episiotomy because it is necessary to allow as much room as possible for the delivery of the shoulder.

The charge nurse is planning nursing assignments on a busy antepartum unit. For which patient does the nurse know that cesarean delivery is contraindicated? A patient with preeclampsia A patient with cephalopelvic disproportion A patient with a confirmed absence of fetal heartbeat A patient with premature rupture of membranes (PROM)

A patient with a confirmed absence of fetal heartbeat Fetal death is a contraindication for cesarean delivery. The surgical risk(s) to the mother (infection, hemorrhage, atelectasis, and anesthesia complications) outweigh the benefit of cesarean delivery in this case.

A nurse receives the shift report when coming onto the antepartum floor. Which patient does the nurse anticipate will require a cesarean delivery? A patient who reports exhaustion A patient with complete placenta previa A patient with genital herpes who has not had an outbreak in 6 months A patient with a history of cesarean delivery with a low vertical uterine incision

A patient with complete placenta previa Complete placenta previa is an indication for cesarean delivery, as this condition may result in hemorrhage during labor.

Which pregnant woman would be advised to go to the hospital? Select all that apply. A pregnant woman with a decrease in the fetal kick count by half A multipara with regular contractions every 5 minutes for 2 hours A pregnant woman who experiences heavy, bright red bleeding A pregnant woman who reports a pink-tinged mucus discharge A primigravida with regular contractions every 15 minutes for 5 hours

A pregnant woman with a decrease in the fetal kick count by half Decreased kick count suggests decreased fetal movement and may be a medical emergency. A multipara with regular contractions every 5 minutes for 2 hours The multipara should report to the hospital if experiencing regular contractions at least 10 minutes apart, for at least 1 hour. A pregnant woman who experiences heavy, bright red bleeding Bleeding that is bright red and not mixed with mucus must be evaluated immediately.

Which premonitory labor sign becomes apparent when changing levels of estrogen and progesterone cause excretion of some of the extra interstitial fluid that accumulates in body tissue during pregnancy? "Bloody show" Braxton Hicks contractions A small weight loss of 1 to 3 lb (2.2 kg to 6.6 kg) Increased clear and nonirritating vaginal secretions

A small weight loss of 1 to 3 lb (2.2 kg to 6.6 kg) A small weight loss of 1 to 3 lb (2.2 kg to 6.6 kg) may occur because changing levels of estrogen and progesterone cause excretion of some of the extra interstitial fluid that accumulates during pregnancy.

Which pregnant women are at risk for PPROM? Select all that apply. A woman who is bearing twins A woman who sits eight hours a day at her job A woman who eats spicy foods at each meal A woman who is depressed over losing her job A woman with a Gardnerella vaginalis infection

A woman who is bearing twins A woman who is depressed over losing her job A woman with a Gardnerella vaginalis infection

A pregnant patient reports breathlessness while at rest and intermittent chest pains. Which patient statements indicate a need for further patient teaching regarding cardiovascular health? Select all that apply. A. "I have gained 10 pounds this week." B. "I only smoke one cigarette per week." C. "Walking on a treadmill provides adequate exercise." D. "I am eating mostly grilled chicken because I crave meat." E. "I am taking vitamins daily, but they are not prenatal vitamins."

A. "I have gained 10 pounds this week." Excessive weight gain can put unnecessary strain on the cardiovascular system and is a cause for concern. B. "I only smoke one cigarette per week." Smoking, in any amount, is a risk factor for cardiovascular disease.

A patient who is HIV positive is about to be discharged from the hospital with her new infant. Which patient statement indicates that further patient teaching is required? A. "I'm glad I'll still be able to breastfeed my baby." B. "It's still possible that I gave my baby HIV, even though I took all my medication." C. "I will go to the pharmacy right now to fill my prescription." D. "Even though I don't feel sick, I should plan on bottle-feeding my baby."

A. "I'm glad I'll still be able to breastfeed my baby." Breast milk is a route of HIV virus transmission and should be avoided completely.

Match the patient symptom to the relevant class of heart disease in a patient who is pregnant. A. Patient with normal vital signs who walks vigorously every morning but experiences nighttime headaches. B. Patient who reports an ability to ride her bike only 5 miles per day instead of the normal 7 due to exhaustion. C. Patient who reports that the household chores cause her to become out of breath. D. Patient with hypertension and heart palpitations on physical examination. 1. Class I 2. Class II 3. Class III 4. Class IV 5. Class V 6. Class VI

A. 1 B. 2 C. 3 D. 4

The nurse expects group beta strep (GBS) testing to be assessed in the patient at which time during pregnancy? A. 35 weeks' B. 12 weeks' C. 20 weeks' D. 28 weeks'

A. 35 weeks' The presence of group beta strep is assessed between 35-37 weeks.

Match the viral infection with its description A. Common infection caused by a herpes virus and is often transmitted by children B. Viral infection transmitted through droplet or direct contact with nasopharyngeal secretions C. Acute viral infection characterized by "slapped-cheeks" appearance D. Known as the chicken pox virus 1. Parvovirus 2. Tuberculosis 3. Toxoplasmosis 4. Rubella 5. Cytomegalovirus 6. Varicella Zoster

A. 5 B. 4 C. 1 D. 6

A woman who is 36 weeks pregnant reports dyspnea at rest and chest pain, and has a blood pressure of 140/90 and a temperature of 98.0 °F on assessment. What should be included in peripartum management of this patient? Select all that apply. A. Bed rest B. Light exercise C. Labor induction D. Antiinfective medication E. Antithrombotic medication

A. Bed rest Patients presenting with signs of class III and/or IV heart disease should be put on bed rest to minimize the metabolic demands on the body. E. Antithrombotic medication Patients presenting with signs of class III and/or IV heart disease should be put on bed rest. Antithrombotic medication may be necessary in patients who are restricted to bedrest to decrease the risk of blood clots due to inactivity.

A patient presents to the Labor and Delivery unit in active labor, and the nurse notes active herpes lesion on her vulva. What type of delivery is best indicated for this patient? A. Cesarean delivery B. Vaginal birth with antibiotic C. Vaginal birth without an antibiotic D. Delaying delivery until lesion is healed

A. Cesarean delivery Cesarean delivery is indicated for a patient in labor with an active herpetic lesion.

The nurse is caring for a patient in labor and has identified a severe deceleration in fetal heart rate. The health care provider has subsequently diagnosed a cord prolapse. Which actions should the nurse perform? A. Continue assessing fetal heart rate. B. Assess the patient's level of anxiety. The nurse should assess the patient's anxiety and C. Ensure that the patient's hips are elevated above the head. Ensuring that the patient's hips are elevated above the D. Palpate the prolapsed umbilical cord to verify presence of pulse. E. Evaluate the family's understanding of the need for immediate delivery.

A. Continue assessing fetal heart rate. B. Assess the patient's level of anxiety. D. Palpate the prolapsed umbilical cord to verify presence of pulse.

A nurse is caring for a pregnant patient with anemia accompanied by cognitive changes and achlorhydria. Which foods should the nurse advise the patient to include in her diet? Select all that apply. A. Fish B. Red meat C. Orange juice D. Green, leafy vegetables

A. Fish Anemia accompanied by neurologic defects and achlorhydria suggests pernicious anemia. The patient should consume fish for iron deficiency anemia, not pernicious anemia. B. Red meat Anemia accompanied by neurologic defects and achlorhydria suggests pernicious anemia. The patient should consume red meat for iron deficiency anemia, not pernicious anemia. D. Green, leafy vegetables Anemia accompanied by neurologic defects and achlorhydria suggests pernicious anemia. The patient should consume food rich in folic acid, including leafy green vegetables.

Which viral infection can result in neonatal brain complications? Select all that apply. A. Herpes infection B. Rubella infection C. Varicella-zoster infection D. Cytomegalovirus infection

A. Herpes infection A herpes infection can cause neonatal encephalitis. B. Rubella infection A rubella infection does not cause neonatal encephalitis.

Which lab result would be an expected finding in patient who is pregnant and has anemia? A. Hgb of 9 grams/dL B. WBC of 6.5 billion cells/L C. Platelet count of 300,000/mcL D. Total iron binding capacity (TIBC) of 350,000 mcg/dL

A. Hgb of 9 grams/dL The hemoglobin would be lower in a patient with anemia.

A nurse is caring for a client in the 3rd stage of labor. Which of the following findings indicate that placental separation has occurred? (SATA) A. Lengthening of umbilical cord B. Swift gush of clear amniotic fluid C. Softening of lower uterine segment D. Appearance of dark blood from vagina E. Fundus is firm on palpation

A. Lengthening of umbilical cord D. Appearance of dark blood from vagina E. Fundus is firm on palpation

While waiting for a cesarean delivery because of cord prolapse, a patient in the Trendelenburg position reports discomfort and asks for her head to be lifted up. The nurse offers to reposition her onto her side with pillows supporting her hips. Which is the nursing rationale for this action? A. Pillows maintain elevation of the hips. B. A side-lying position relieves pressure on the patient's back. C. The head should not be lifted to prevent orthostatic hypotension. D. A side-lying position reduces the risk of precipitate birth while awaiting a cesarean section.

A. Pillows maintain elevation of the hips.

An HIV-positive patient is 34 weeks' pregnant. Delivering via cesarean section at 38 weeks can help prevent which complications in the neonate? Select all that apply. A. Pneumonia B. Septicemia C. Meningitis D. Osteomyelitis E. Hydrocephalus F. Encephalopathy

A. Pneumonia Pneumonia is a bacterial infections that a HIV-positive newborn may develop. B. Septicemia Septicemia is a bacterial infections that a HIV-positive newborn may develop. C. Meningitis Meningitis is a bacterial infections that a HIV-positive newborn may develop. D. Osteomyelitis Osteomyelitis is a bacterial infections that a HIV-positive newborn may develop.

A patient who is pregnant complains of sensitivity to light, pain in the knees and wrists, and a facial rash. The patient is at risk for which conditions that may affect the health of the fetus? Select all that apply. A. Preeclampsia B. Hypertension C. Renal complications D. Fetal hydantoin syndrome E. Preterm rupture of membranes

A. Preeclampsia Preeclampsia may occur early and be severe for a patient with lupus, as characterized by photosensitivity, pain in the joints, and a facial rash. B. Hypertension A patient with signs of lupus is at risk for premature birth and potential pregnancy loss due to hypertension. C. Renal complications A patient with signs of lupus is at risk for premature birth and potential pregnancy loss due to renal complications. E. Preterm rupture of membranes A patient with signs of lupus is at risk for preterm rupture of membranes, putting the health of the fetus at risk.

A child is born HIV positive. Which is the most likely manner the child contracted the disease? A. Vaginal birth B. Skin-to-skin contact C. Cesarean section birth D. Transmission of HIV through the placenta

A. Vaginal birth Vaginal birth is a route of the transmission of HIV during pregnancy.

The nurse is caring for a 10-minute-old newborn. The nurse notes RR 34, HR 122, flexed extremities, and lusty cry. Which health care provider order would the nurse implement first? Administer Vitamin K Assess blood pressure Obtain serum glucose level Apply erythromycin ointment

Administer Vitamin K Newborns are born without clotting factors. Vitamin K is necessary for clotting factor production to prevent bleeding disorders. The nurse would administer the Vitamin K first to prevent excessive bleeding.

The nurse is preparing to administer required medications to a newborn after a vaginal delivery to prevent eye infection. Which action does the nurse take? Administer intravenous ampicillin Administer erythromycin ointment Administer vitamin K intramuscularly Wash the newborn's eyes with mild soap and water

Administer erythromycin ointment Erythromycin ophthalmic ointment is administered to newborns to prevent infection.

The postpartum nurse is caring for a patient who experienced a cesarean delivery and whose vertical skin incision is closed with sutures and surgical glue. Which step should the nurse take to ensure the patient's comfort during the assessment? Assist the patient to extend her legs prior to palpating the fundus. Avoid palpating the area around the incision during assessment. Decrease the frequency of assessments to allow for patient rest. Administer the patient's pain medication before fundal assessment.

Administer the patient's pain medication before fundal assessment. Patients with vertical skin incisions often experience greater discomfort during fundal assessment than patients with Pfannenstiel incisions. Therefore, administering pain medication before fundal assessment helps ensure patient comfort and is a step the nurse should take during assessment.

A nurse examines a newborn 3 hours after birth. Which assessment findings are expected during this period? Select all that apply. Alert Interest in feeding Decreased activity Decreased heart rate Active alert/Fully awake Decreased temperature

Alert The newborn is alert and responsive to the environment during the first period of reactivity. Active alert/Fully awake The newborn is active alert and wide awake during the first period of reactivity. Decreased temperature The newborn's temperature decreases during the first period of reactivity.

The mother of a newborn child enquires about having her child's hearing checked. What is the best response by the nurse? All newborns should be screened before 1 year of age. At 6 months you can return and we will screen your infant. Your newborn will be screened at the four-month well-baby visit. All newborns should be screened for hearing before one month of age.

All newborns should be screened for hearing before one month of age. Stating that the child will be screened for hearing before one month of age is accurate and therefore this is the best response for the nurse to make.

An infant born in a birthing center is being transferred to a regional neonatal intensive care unit because of respiratory distress. Which nursing action best promotes parent-infant attachment? Encouraging the parents to call their infant by name Allowing the parents to hold their infant before departure Giving the parents a picture of their infant in the intensive care unit Instructing the parents to contact the neonatal intensive care unit daily

Allowing the parents to hold their infant before departure (Because seeing and touching the newborn infant are species-specific behaviors for human attachment, allowing the parents to hold the infant will promote bonding. Although encouraging the parents to call the infant by name is a useful action, holding and touching will promote bonding more effectively. After touching and holding, having a picture of their infant in the intensive care unit contributes most to bonding. Actual holding and touching promote bonding more than just hearing about the infant's progress.)

The nurse is caring for a patient with an episiotomy 4 hours after delivery. While examining the wound, the nurse notices a hard, turgid area alongside the incision. Which is the next step in nursing management? Massage the hardened area Take the patient's temperature Apply an ice pack to the perineum Apply a warm, damp towel to the perineum

Apply an ice pack to the perineum Ice should first be applied to the site to promote vasoconstriction. With less blood flowing to the area of the incision, inflammation may decrease. The nurse should follow up by notifying the health care provider.

Which actions are key prenatal nursing interventions for the pregnant patient addicted to heroin? Select all that apply. Avoid showing general interest and concern when communicating with the patient. Ask the woman about stressors in her life that may be contributing to her substance abuse. Coordinate care among various service providers such as group therapy and prenatal classes. Praise any attempts at abstinence, and encourage the expectant mother to try again if she relapses. At each antepartum visit, review the current status of substance use, social service needs, education needs, and compliance with treatment referrals.

Ask the woman about stressors in her life that may be contributing to her substance abuse. Coordinate care among various service providers such as group therapy and prenatal classes. Praise any attempts at abstinence, and encourage the expectant mother to try again if she relapses. At each antepartum visit, review the current status of substance use, social service needs, education needs, and compliance with treatment referrals.

The nurse assesses a neonate born to a mother with a history of heroin use during pregnancy. Which is a priority assessment for the nurse to complete? Observe for flexed upper extremities when startled. Assess for tremors when unwrapping the neonate. Observe for a dark black, viscous stool in the diaper. Observe for decreased weight loss from the previous day.

Assess for tremors when unwrapping the neonate.

Nursing Interventions During Fourth Stage of Labor

Assess maternal vitals, fundus and lochia every 15 minutes for 1st hour then according to facility protocol Massage uterine fundus and/or administer oxytocics to maintain uterine tone and prevent hemorrhage Encourage voiding to avoid bladder distention Promote paternal-newborn bonding

A woman in preterm labor is receiving intravenous magnesium sulfate. Which are the nurse's priority assessments? Select all that apply. Assess respirations Monitor urine output Check the capillary refill Assess for deep tendon reflexes Check the level of consciousness

Assess respirations Monitor urine output Assess for deep tendon reflexes

Which assessments are included when the nurse assesses the newborn's cardiorespiratory status? Select all that apply. Assess the airway Auscultate heart sounds Inspect the umbilical cord Inspect extremity skin color Determine capillary refill time

Assess the airway The nurse should assess the airway patency of the newborn to determine cardiorespiratory status. The airway may be blocked with mucus and should be suctioned. Auscultate heart sounds The nurse should assess the heart sounds of the newborn to determine cardiorespiratory status. Abnormal heart sounds, like murmurs, may indicate abnormalities in cardiovascular adaptation. Determine capillary refill time The nurse should assess the capillary refill time of the newborn to determine cardiorespiratory status. Delayed capillary refill may indicate cardiac anomalies.

During the newborn assessment, the nurse notes poor muscle tone, low temperature, and lethargy. Which nursing action does the nurse take? Offer a feeding Administer glucose water Encourage skin-to-skin contact Assess the newborn's blood glucose

Assess the newborn's blood glucose Poor muscle tone, low temperature, and lethargy are signs of hypoglycemia. The nurse should assess the blood glucose and send findings to the health care provider.

The nurse is caring for a client who has a newborn with a neurologic impairment. What is the most important nursing action at this time? Assisting the client with the grieving process Performing frequent neurologic assessments of the newborn Arranging for social services to discuss possible placement of the newborn Obtaining a prescription for an antidepressant to help the client cope with the depressing news

Assisting the client with the grieving process (Grieving is expected and necessary whenever a newborn is born less than healthy. More information is needed to conclude that frequent neurologic assessments are warranted; the frequency of assessments depends on the severity and type of the neurologic problem. Arranging for social services to discuss possible placement of the newborn may be done later; however, it is not the priority at this time. Obtaining a prescription for an antidepressant to help the client cope with the depressing news could result in a delay in the client's ability to actively participate in dealing with feelings.)

Which series of activities best reflects the motor development of an infant from 1 month of age to 4 months? Select all that apply. At 4 months, the rooting reflex disappears. At 1 month, the child will hold a fist tightly clenched. At 3 months the child will be able to crawl a minimal distance. At 3 months the child will be able to lift the head 90 degrees from a prone position. At 4 months the child can hold the head steadily erect in a sitting position.

At 4 months, the rooting reflex disappears. There are several primitive reflexes that disappear after the first year of life. The Moro reflex and the rooting reflex will disappear by 4 months of age. At 1 month, the child will hold a fist tightly clenched. At one month the child will keep the hand clenched and this will start to change by 2 months of age. At 3 months the child will be able to lift the head 90 degrees from a prone position. The muscle strength in the baby's head by 3 months of age will allow for lifting to 90 degrees in a prone position. At 4 months the child can hold the head steadily erect in a sitting position. At four months a child has motor development sufficient enough to hold the head erect in the sitting position.

A pregnant woman presents with clumpy, white vaginal discharge, vulvar pruritus, and burning. Which treatment would the nurse expect for this patient? A. An antibiotic B. An antifungal medication C. An antiemetic medication D. An antiparasitic medication

B. An antifungal medication Clumpy white vaginal discharge with vulvar pruritus and burning are associated with candidiasis and should be treated with the appropriate antifungal such as miconazole.

A neonate is born to a mother with HIV. Which test is a priority to perform? A. Platelet count B. CD4 lymphocyte count C. Red blood cell (RBC) count D. White blood (WBC) cell count

B. CD4 lymphocyte count Monitoring CD4 count is critical assessment in HIV patients.

A patient who is pregnant is diagnosed with a viral illness. Which viruses can lead to low birth weight? Select all that apply. A. HIV B. CMV C. Rubella D. Influenza E. Varicella Zoster

B. CMV CMV infection is associated with fetal growth restriction and may result in a low birth weight. C. Rubella Rubella infection is associated with fetal growth restriction and may result in a low birth weight. E. Varicella Zoster Varicella Zoster infection is associated with fetal growth restriction and may result in a low birth weight.

Which symptom warrants immediate assessment by the nurse if present in a patient who is pregnant and diagnosed with cardiovascular disease? A. Headaches B. Chest pain with walking up stairs C. Shortness of breath with exercise D. Swelling of the feet when standing for long periods

B. Chest pain with walking up stairs Class IV classification of heart disease represents high risk for mother and child and is characterized by the inability to perform physical activity without discomfort.

The nurse reviews the health history of an infant. The nurse notes the infant's birth weight to be 3.5 lbs, with a diagnoses of spina bifida and an abnormal heart rate. Which maternal condition may best explain the findings? A. HIV B. Epilepsy C. Chronic hypertension D. Systemic lupus erythematosus

B. Epilepsy Folate deficiency as a result of antiepileptic medication taken by an epileptic mother can cause fetal hydantoin syndrome, which includes craniofacial abnormalities, neural tube defects, limb reduction defects, growth restriction, intellectual disability, and cardiac anomalies.

A patient who is 11 weeks pregnant has a hemoglobin level of 9 g/dL. She reports lethargy and a craving for bread. The nurse determines that which may be a potential fetal complication? A. Hypoglycemia B. Low birth weight C. Hyperbilirubinemia D. Respiratory distress syndrome (RDS)

B. Low birth weight Low birth weight is associated with iron deficiency anemia.

Which nursing intervention should be included in the care of a woman with lupus during her first trimester of pregnancy? A. Monitor for liver function B. Monitor for lupus remission C. Provide patient education on remission of lupus D. Administer subcutaneous heparin for thrombosis prevention

B. Monitor for lupus remission Pregnancy can worsen progression of disease and the disease should be carefully monitored during pregnancy to avoid serious complications.

Which physiological change of pregnancy increases the risk of anemia in pregnancy? A. Platelet count increases B. Plasma volume increases C. The ferritin level increases D. White blood cell volume increases

B. Plasma volume increases Plasma volume increase during pregnancy, causing a dilutional anemia.

A woman at 30 weeks' gestation has a blood pressure of 160/90, which has been consistently elevated for 15 weeks. The woman should be monitored for which additional condition that may result? A. Epilepsy B. Preeclampsia C. Fetal hydantoin syndrome D. Systemic lupus erythematosus

B. Preeclampsia Preeclampsia may develop in patients with chronic hypertension and can lead to fetal growth restriction.

A nurse is caring for a client who is in the transition stage of labor and reports that she needs to have a bowel movement with the peak of contractions. Which of the following is an appropriate nursing intervention? A. Assist the client to the bathroom B. Prepare for an impending delivery C. Prepare to remove fecal impaction D. Encourage client to take deep, cleansing breath

B. Prepare for an impending delivery rationale: urge for BM indicates fetal descent

The nurse is caring for a patient who has just been admitted to the labor and delivery unit. The patient suddenly grips the side rails and yells loudly, and the nurse can see the infant rapidly crowning. Which action should the nurse take first? A. Contact the health care provider. B. Reposition the patient to a side-lying position. C. Administer oxygen to the patient through a face mask. D. Reposition the patient so the hips are elevated above the head

B. Reposition the patient to a side-lying position.

For which type of anemia does a family history of the disease increase the patient's risk of acquiring? A. Aplastic anemia B. Sickle cell anemia C. Pernicious anemia D. Iron deficiency anemia

B. Sickle cell anemia Sickle cell anemia is a recessively inherited trait for which a family history increases risk.

A pregnant woman with systemic lupus erythema is at risk for which condition? A. Ectopic pregnancy B. Spontaneous abortion C. Multiple gestation pregnancy D. Gestational trophoblastic disease

B. Spontaneous abortion Spontaneous abortion is associated with systemic lupus erythema in the first trimester.

A neonate born at 35 weeks' gestation has Apgar scores of 8 and 9. At 4 hours of age the newborn begins to experience respiratory distress, has a below-normal temperature in a warm environment, and has a low blood glucose level. What problem does the nurse suspect? Hypoglycemia Bacterial sepsis Cocaine withdrawal Meconium aspiration

Bacterial sepsis (Preterm neonates react to infection with respiratory distress and subnormal temperatures. Although hypothermia is one sign of hypoglycemia, the newborn is not exhibiting other signs, such as tremors and lethargy. The data do not indicate that meconium was present at birth. Four hours of age is too early for signs of cocaine withdrawal to occur.)

`Nursing Assessment During Second Stage of Labor

Begins with complete dilation and effacement Increase in bloody show Pushing efforts by client Vitals every 5-30 minutes FHR every 15 minutes Assess for perineal lacerations that occur as fetus is expulsed, defined in terms of depth

Culturally Competent Care: European Americans

Birth is public concern; focus on technology; partner expected to be involved; provider seen as head of health care team

A newborn of a mother with diabetes is jittery and hypotonic. The nurse should complete which assessment first in the neonate? Respiratory rate Axillary temperature Blood glucose level Readiness to nipple feed

Blood glucose level

Which finding in a newborn whose temperature over the last 4 hours has fluctuated between 98.0° F (36.7° C) and 97.4° F (36.3° C) would be considered critical? Respiratory rate of 60 breaths/min White blood count greater than 15,000 mm3 Serum calcium level of 8 mg/dL (2 mmol/L) Blood glucose level of 36 mg/dL (3.8 mmol/L)

Blood glucose level of 36 mg/dL (3.8 mmol/L) (Instability of the newborn's temperature is an indication of hypoglycemia. A glucose level below 40 mg/dL (1.7 mmol/L) does not provide enough energy to maintain the body temperature at a normal level. A serum calcium level of 8 mg/dL (2 mmol/L), respiratory rate of 60 breaths/min, and a white blood cell count greater than 15,000 mm3 are all normal findings and do not affect body temperature.)

A neonate with a history of asphyxia from birth is hypotonic, has audible grunting, and has severe subcostal retractions. Which findings should the nurse report to the health care provider? Select all that apply. Blood pH level of 7.1 Substernal retractions Respiratory rate of 56 bpm Pulse oximeter reading of 88% on room air Cyanosis in the hands and feet of the neonate

Blood pH level of 7.1 Substernal retractions Pulse oximeter reading of 88% on room air

An infant is admitted to the nursery after a difficult shoulder dystocia vaginal birth. Which condition should the nurse carefully assess this newborn for? Facial paralysis Cephalhematoma Brachial plexus injury Spinal cord syndrome

Brachial plexus injury (Brachial plexus paralysis (Erb-Duchenne palsy) is the most common injury associated with dystocia related to a shoulder presentation; it is caused by pressure and traction on the brachial plexus during the birth process. The newborn's face is not involved with a shoulder presentation. Cephalhematoma is a soft-tissue injury of the head and is not related to shoulder dystocia. Spinal cord syndrome is associated with a breech presentation and is not related to shoulder dystocia.)

Relaxation Measures During Labor

Breathing Effleurage Distraction (focal point or imagery)

The nurse is assessing a term newborn. Which sign should the nurse report to the pediatric primary healthcare provider? Temperature of 97.7° F (36.5° C) Pale-pink to rust-colored stain in the diaper Heart rate that decreases to 115 beats/min Breathing pattern with recurrent sternal retractions

Breathing pattern with recurrent sternal retractions (A breathing pattern with recurrent sternal retractions is indicative of respiratory distress; the expected pattern is abdominal with synchronous chest movement. A temperature of 97.7° F (36.5° C) is within the expected range of 97.6° F (36.4° C) to 99° F (37.2° C) for a newborn. Pale-pink to rust-colored staining in the diaper is caused by uric acid crystals from the immature kidneys; it is a common occurrence. A decrease in heart rate to 115 beats/min is within the expected range of 110 to 160 beats/min for a newborn.)

The nurse is assessing the feeding of a bottle-fed newborn and notes a slightly distended abdomen and fussiness right after feeding. The nurse suspects excessive air swallowing and makes which suggestions to the parents regarding feeding? Provide less frequent feedings Burp the newborn during feedings Double the water added to the formula Position the newborn in a semi-Fowler's position Seek medical attention if symptoms do not improve Hold the bottle up so that no air is allowed into the nipple .

Burp the newborn during feedings Burping during feedings decompresses the stomach and prevents excessive gas in the stomach. Position the newborn in a semi-Fowler's position Positioning the newborn in an upright position facilitates digestion and passage of formula through the GI tract. Seek medical attention if symptoms do not improve Abdominal distention may indicate a bowel obstruction. If symptoms do not improve, the parent should seek medical attention. Hold the bottle up so that no air is allowed into the nipple Keeping the nipple full of milk prevents air swallowing.

The nurse is assessing the breastfeeding of a new mother and notes the newborn "spits up" immediately after feedings. Which does the nurse know could help? Lying the newborn prone after feedings. Burping the newborn halfway through feedings. Offering the newborn a cold bottle instead of breast milk. Providing the newborn with shorter, more frequent feedings.

Burping the newborn halfway through feedings. Burping the newborn halfway through the feeding will decompress the stomach and prevent regurgitation.

A nurse is educating a pregnant patient on sexually transmitted infections. Which statement is best for the nurse to make? A. "You don't need to use condoms now that you are already pregnant." B. "Prenatal testing for sexually transmitted infections is not necessary." C. "Sexually transmitted infections can pose serious health risks for you and your pregnancy." D. "Sexually transmitted infections do not cross the placenta and do not pose a risk to your baby."

C. "Sexually transmitted infections can pose serious health risks for you and your pregnancy." Patient education during pregnancy should include the health risks posed by sexually transmitted infections.

A nurse is caring for a client and her partner during the 2nd stage of labor. The client's partner asks the nurse to explain how he will know when crowning occurs. Which of the following is an appropriate response? A. "The placenta will protrude from the vagina" B. "Your partner will report a decrease in the intensity of contractions" C. "The vaginal area will bulge as the baby's head appears" D. "Your partner will report less rectal pressure"

C. "The vaginal area will bulge as the baby's head appears"

The nurse administered magnesium sulfate to a woman experiencing preterm labor following a health care provider order. Which assessment would concern the nurse? A. Urine output of 50 mL B. Oxygen saturation of 95% C. 10 respirations per minute D. Bronchial sounds heard over the body of the sternum

C. 10 respirations per minute

Which patient is most at risk for developing chronic hypertension during pregnancy? A. A 25-year-old who drinks heavily and does not exercise B. A 42-year-old patient in good health with three previous children C. A 35-year-old patient who weighs 350 pounds and walks daily D. A 22-year-old patient who weighs 100 pounds and has suffered two miscarriages

C. A 35-year-old patient who weighs 350 pounds and walks daily Chronic hypertension is most often seen in patients who are obese or have diabetes. The patient's weight puts them at a higher risk for development of the condition.

A 22-year-old nulligravida is trying to conceive. On assessment, the patient has a calculated BMI of 22, resting heart rate of 75 bpm, blood pressure of 160/90, and temperature of 98.7 °F. What is the priority action to minimize potential cardiovascular problems prior to conceiving? A. Administration of an antibiotic B. Patient education regarding weight loss C. Administration of antihypertensive medication D. Recommendation that the patient waits until she is older

C. Administration of antihypertensive medication The patient's high blood pressure is a concern and should be controlled before the patient becomes pregnant, either through diet and exercise or through administration of antihypertensive medication.

A woman at 34 weeks' gestation presents with painful uterine contractions, lower back pain, and bloody vaginal discharge. Additionally, the woman has a temperature of 102° F and is noted to be at 2 cm dilated. Which action should the nurse take first? A. Assess for infection. B. Provide emotional support. C. Place the woman on bed rest. D. Evaluate fetus for distress, size, and maturity.

C. Place the woman on bed rest

A pregnant patient with untreated bacterial vaginosis should be educated about which considerations? Select all that apply. A. Thrush may develop in newborns. B. Infection may require an intravenous medication. C. Preterm labor and birth is a possible adverse pregnancy outcome. D. Postpartum endometritis is a possible adverse pregnancy outcome. E. An intraamniotic infection is a possible adverse pregnancy outcome.

C. Preterm labor and birth is a possible adverse pregnancy outcome. This is appropriate because a vaginosis infection can lead to adverse pregnancy outcomes, including preterm labor and birth. D. Postpartum endometritis is a possible adverse pregnancy outcome. This is appropriate because a vaginosis infection can lead to adverse pregnancy outcomes, including postpartum endometritis. E. An intraamniotic infection is a possible adverse pregnancy outcome. This is appropriate because a bacterial vaginosis can lead to adverse pregnancy outcomes, including an intraamniotic infection.

A woman tests positive for group beta strep prior to a vaginal birth. The nurse would anticipate which orders to be included in this patient labor? A. There is no change in patient treatment B. The patient should be given zidovine during labor C. The patient should be given penicillin prior to labor D. The patient should be given ceftriaxone prior to labor

C. The patient should be given penicillin prior to labor Penicillin is the first-line antibiotic for group beta strep (GBS).

The labor and delivery nurse is monitoring a laboring patient and notes the presence of variable decelerations on the fetal monitor tracing. Which finding should the nurse expect based on this observation? A. Pressure on the fetal head B. Normal fetal heart activity C. Umbilical cord compression D. Uteroplacental insufficiency

C. Umbilical cord compression

During which stages of pregnancy can a mother pass genital herpes to her newborn? Select all that apply. A. Gestation B. Conception C. Vaginal labor D. Cesarean section E. Rupture of the membranes

C. Vaginal labor A mother with active lesions, especially primary lesions, can pass genital herpes to her child during vaginal labor. E. Rupture of the membranes Genital herpes can be passed from mother to baby after rupture of membranes, when the active virus ascends from active lesions.

The antepartum nurse is caring for a couple whose amniocentesis results indicated a positive neural tube defect. The couple is visibly upset and concerned about the future of the pregnancy. Which nursing care is appropriate for this couple? Select all that apply. A. Allow the couple time to grieve the loss of the normal infant they had anticipated. B. Provide the couple with contact information for available counseling services. C. Caution the couple against terminating the pregnancy before determining the nature of the defect. D. Explain to the couple that the decision to continue or to terminate the pregnancy is entirely their choice. E. Ask for clarification when the couple states they are "not making a decision" as to whether to continue the pregnancy.

CORRECT A. Allow the couple time to grieve the loss of the normal infant they had anticipated. The couple who has received abnormal fetal diagnostic test results should be given time to grieve for the expected normal infant. B. Provide the couple with contact information for available counseling services. The couple should be given resources necessary to process the shock of the abnormal test result and to grieve the loss of the expected normal infant. D. Explain to the couple that the decision to continue or to terminate the pregnancy is entirely their choice. Perinatal counseling is nondirective. The couple should be informed that the decision to terminate or continue the pregnancy is entirely their choice.

A patient reports frustration at the health care provider's decision not to perform all of the diagnostic tests she wishes to have. Which patient education is appropriate? Select all the apply A. Tell the patient that her concerns will be reported to the provider. B. Explain to the patient that some abnormalities can be detected by more than one test. C. Inform the patient that she has the right to seek treatment elsewhere if she is not satisfied. D. Explain to the patient that even if all tests were normal, it would not rule out every abnormality. E. Inform the patient that each procedure carries risks to both patient and infant, and that risk increases when multiple procedures are performed.

CORRECT B. Explain to the patient that some abnormalities can be detected by more than one test. Explaining the overlap between diagnostic test capabilities helps the patient better understand the purpose and efficacy of fetal diagnostic testing. D. Explain to the patient that even if all tests were normal, it would not rule out every abnormality. Informing the patient of the baseline risk that exists regardless of normal test results helps the patient better understand the purpose of fetal diagnostic testing. E. Inform the patient that each procedure carries risks to both patient and infant, and that risk increases when multiple procedures are performed. Explaining the risks of diagnostic procedures helps the patient better understand the risks and advantages of fetal diagnostic testing.

A patient with no family history of genetic abnormalities asks the nurse if there may still be the need for prenatal genetic counseling. Which patient statement indicates that genetic counseling may be necessary? A."My partner is 14 years older than me." B. "I was adopted by my biological mother's sister." C. "One of my children went deaf in one ear after an ear infection." D. "My family and I practice a religion that does not allow outside marriage."

CORRECT D. "My family and I practice a religion that does not allow outside marriage." Patients who are members of groups that are isolated by geography, religion, or culture are at increased risk for autosomal recessive traits.

A woman typically consumes 1300 calories per day prior to becoming pregnant. How many calories should the patient consume during her pregnancy? ________

CORRECT 1600 A pregnant woman needs an additional 300 calories during pregnancy and 500 additional calories for lactation postpartum each day. Therefore, a woman who consumed 1300 calories per day prior to pregnancy would need to consume 1600 calories per day during her pregnancy and 1800 calories per day during postpartum lactation.

Match the prenatal education classes with their appropriate description. A. C-section B. Postpartum C. Breastfeeding D. Childbirth preparation 1. Class covers newborn care 2. Class covers what to expect after delivery 3. Class covers surgical technique and what to expect after delivery 4. Class covers exercises during pregnancy

CORRECT 3. Class covers surgical technique and what to expect after delivery Class covers exercises during pregnancy B. 2. Class covers what to expect after delivery C. 1. Class covers newborn care D. 4. Class covers exercises during pregnancy

A patient with a family history of spina bifida reports reluctance to attempt conception without knowing the risk of having an infant with spina bifida. Which response from the nurse is appropriate? A. "Development of spina bifida depends on more than just genetic factors." B. "The only way to screen for spina bifida is through testing after conception." C. "Spina bifida is not affected by genetics; therefore, the risk cannot be determined." D. "Genetic counseling can determine your exact risk of having a child with spina bifida."

CORRECT A. "Development of spina bifida depends on more than just genetic factors." Spina bifida is caused by genetic factors, as well as by environmental influences, such as inadequate maternal intake of folic acid.

A patient has not felt fetal movement in several days and is scheduled for an ultrasound. Which statement by the nurse helps explain the necessity of the ultrasound? A. "The ultrasound will let us see your baby's heartbeat." B. "The ultrasound can show us where your baby is located." C. "The ultrasound will help us be certain of your baby's gestational age." D. "With the ultrasound, we can measure how much your baby has grown since your last appointment."

CORRECT A. "The ultrasound will let us see your baby's heartbeat." Absence of fetal movement for several days may indicate multiple adverse events, including fetal death. Ultrasound is used to confirm fetal viability, which is the indication that applies to this patient.

Which patient may be a carrier for an X-linked recessive trait? A. A patient whose son is colorblind. B. A patient whose father was born with cystic fibrosis. C. A patient whose daughter was born with Turner syndrome. D. A patient whose brother was born with Trisomy 21 (Down syndrome).

CORRECT A. A patient whose son is colorblind. Colorblindness is an X-linked recessive trait that is commonly displayed in males. Because this patient's son expresses the recessive gene, this patient may be a carrier for an X-linked recessive trait.

Which patient would benefit from prenatal genetic counseling? A. A patient with sickle cell disease (SCD) B. A patient who is having difficulty conceiving C. A patient with history of two multifetal pregnancies D. A patient with history of miscarriage due to polyploidy

CORRECT A. A patient with sickle cell disease (SCD) Sickle cell disease (SCD) is an autosomal recessive trait that can severely impact a patient's life. This patient would benefit from genetic counseling at any point in the perinatal care period.

The nurse understands that which intervention is most appropriate for a patient with Rh-negative blood who is unsensitized and just received percutaneous umbilical blood? A. RhoGAM administration B. Transvaginal ultrasound C. Transabdominal ultrasound D. Anticoagulant administration

CORRECT A. A. RhoGAM administration A patient who is Rh-negative and unsensitized should be given RhoGAM following percutaneous umbilical blood sampling (PUBS) to prevent Rh sensitization.

A patient with a family history of developmental disability refuses a recommended amniocentesis without explanation. Which patient education is appropriate? Select all that apply A. Affirm that the patient has the right to refuse any procedure. B. Explain to the patient that amniocentesis can be used to diagnose developmental disabilities. C. Inform the patient that refusing the amniocentesis may hinder the effectiveness of prenatal care. D. Inform the patient that prenatal diagnosis can give the family more time to prepare for a child with special needs. E. Remind the patient that the health care provider ordering the procedure may simply order a test besides amniocentesis if the patient refuses.

CORRECT A. Affirm that the patient has the right to refuse any procedure. Affirming the patient's right to refuse treatment provides reassurance to the patient and respects the patient's autonomy. B. Explain to the patient that amniocentesis can be used to diagnose developmental disabilities. Explaining the purpose of amniocentesis as it relates to the patient's history provides the patient with context necessary to make an informed decision. D. Inform the patient that prenatal diagnosis can give the family more time to prepare for a child with special needs. Informing the patient of the potential benefits of amniocentesis as it relates to the patient's history provides context to address a potential knowledge deficit that may have influenced the patient's refusal.

The antepartum nurse is caring for a patient with a family history of neural tube defects. The patient declines genetic counseling after listening to the health care provider's explanation. Which nursing intervention is appropriate for this patient? A. Continue to the next part of the visit. B. Inform the patient that declining genetic counseling may lead to inadequate preparation if the infant is born with a defect. C. Provide the patient with printed material regarding genetic counseling to take home should the patient have a change of mind. D. Explain to the patient that the test must be done because a family history of neural tube defects indicates a need for genetic counseling.

CORRECT A. Continue to the next part of the visit. Whether to receive genetic counseling is ultimately the patient's decision. The nurse should acknowledge the patient's choice and continue with the visit.

The nurse should encourage the pregnant woman to receive which vaccinations during pregnancy? Select all that apply. A. Flu B. Polio C. MMR D. TDAP E. Hepatitis B

CORRECT A. Flu Flu vaccinations are advised during pregnancy, as the flu often has more serious manifestations in pregnant woman and can result in preterm birth. D. TDAP TDAP vaccinations are encouraged during pregnancy and can provide protection for the baby against pertussis

Why should the pregnant patient be encouraged to get adequate amounts of folic acid during pregnancy? A. Folic acid prevents birth defects. B. Folic acid prevents preterm labor. C. Folic acid increases iron absorption. D. Folic acid helps maintain the pregnancy.

CORRECT A. Folic acid prevents birth defects. It is important for a pregnant patient to take adequate folic acid to prevent neural tube defects specific to the spinal cord and the brain.

The nurse is explaining the duties of the support person during labor. Which duties should be included in teaching? Select all that apply. A. Keeping patient calm B. Keeping track of contractions C. Interpretation of fetal monitoring D. Administration of pain medication E. Ensuring physical and emotional comfort

CORRECT A. Keeping patient calm The support person should help with breathing and relaxation techniques during labor to keep the patient calm and focused. B. Keeping track of contractions The support person should help time contractions during labor. This will help the woman to know when to come to the hospital for labor and birth. E. Ensuring physical and emotional comfort The support person should help with comfort measures during labor and assist with emotional support of the patient. This will help the woman to have less anxiety and promote relaxation during labor.

A patient at 34 weeks' gestation and her partner report being nervous about the upcoming delivery and baby care. Which classes should the nurse recommend? Select all that apply A. Newborn care B. Prenatal exercise C. Early prenatal care D. Preconception care E. Childbirth preparation

CORRECT A. Newborn care Newborn care class should be recommended as they can teach expecting parents how to care for the infant. Adequate preparation and education can quell anxiety and fear. E. Childbirth preparation Childbirth preparation class should be recommended as adequate education regarding preparation for birth can help decrease the anxiety of the couple.

The nurse is assessing a pregnant patient in the women's health clinic. When planning care for the patient, the nurse recognizes which information supports the need for fetal diagnostic procedures to be performed? Select all that apply. A. Parity B. Maternal age C. Fetal movement D. Nondrug allergies E. History of cesarean delivery

CORRECT A. Parity Grand multiparity (>5 pregnancies) indicates the need for fetal diagnostic procedures; therefore, parity is a relevant indicator of whether fetal diagnostic procedures should be performed. B. Maternal age Maternal age of <16 or >35 years indicates the need for fetal diagnostic procedures; therefore, age is a relevant indicator of whether fetal diagnostic procedures should be performed. C. Fetal movement Decrease in or absence of fetal movement indicates the need for fetal diagnostic procedures; therefore, fetal movement is a relevant indicator of whether fetal diagnostic procedures should be performed.

A 38-year-old patient declines prenatal diagnostic testing due to a lack of family history of genetic or chromosomal abnormalities. Which nursing education is appropriate for this patient? A. The patient's age increases the risk of having a child with an abnormality. B. Prenatal diagnostic testing is recommended even if no risk factors are present. C. If no family history of abnormalities is present, prenatal diagnostic testing is not recommended. D. The patient should receive prenatal diagnostic testing to screen for potential recessive gene abnormalities.

CORRECT A. The patient's age increases the risk of having a child with an abnormality. Advanced maternal age increases the risk of chromosomal trisomy, such as Trisomy 21 (Down syndrome).

What are the goals of perinatal education classes? Select all that apply A. To decrease anxiety related to childbirth B. To teach parents how to perform CPR for their children C. To provide knowledge related to the labor process and birth D. To increase knowledge of comfort relief measures during labor E. To teach the expectant families how to parent and discipline their children

CORRECT A. To decrease anxiety related to childbirth Decreasing anxiety related to childbirth is a goal of perinatal education classes. C. To provide knowledge related to the labor process and birth Providing knowledge related to the labor process and birth is a goal of perinatal education classes. D. D. To increase knowledge of comfort relief measures during labor Increasing knowledge of comfort relief measures during labor is a goal of perinatal education classes.

A pregnant patient should be instructed to include which information in her birth plan? Select all that apply. A. Activity during labor B. Methods of pain relief C. Timing of contractions D. Fetal monitoring interpretation E. Positioning during pushing/birth F. Support persons present during labor/birth

CORRECT A.Activity during labor Activity during labor should be included in a birth plan. This informs the health care provider of the woman's preference for ambulation during labor. Allowing women to have choices increases their confidence and decreases anxiety related to labor and birth. B. Methods of pain relief Methods of pain relief should be included in a birth plan. This informs the health care provider of the woman's preference for pain relief, such as natural comfort measures, IV pain medication, and/or epidural anesthesia. Allowing women to have choices increases their confidence and decreases anxiety related to labor and birth. E. Positioning during pushing/birth Positioning during pushing/birth should be included in a birth plan. This informs the health care provider of the woman's preference for positions while pushing. Allowing women to have choices increases their confidence and decreases anxiety related to labor and birth. F. Support persons present during labor/birth Support persons present during labor/birth should be included in a birth plan. This will allow for the woman to have more control and less anxiety during labor and birth.

A woman visits the clinic for preconception counseling. Which should the nurse assess? Select all that apply. A. Nutrition B. Maternal health C. Fetal heart tones D. Alcohol/drug use E. Current medications F. Fundal height measurement

CORRECT A.Nutrition Nutrition, especially folic acid intake, is assessed. Folic acid should be consumed at least one month prior to conception to prevent neural tube defects. B. Maternal health Maternal health should be assessed, as well as chronic medical problems that can affect the ability to conceive and/or the pregnancy. D. Alcohol/drug use Alcohol/drug use should be assessed before conception as these are harmful to the developing fetus and to the mother. E. Current medications Current medications should be assessed as medications can be teratogenic to a developing fetus.

A patient who is 10 weeks pregnant feels uncomfortable having a transvaginal ultrasound and asks the nurse if there is any way she could have a transabdominal ultrasound instead. Which is the appropriate response from the nurse? Select all that apply A. "We can perform a transabdominal ultrasound if your bladder is full." B. "If it makes you feel more comfortable, you can insert the probe yourself." C. "We would not be able to perform a transabdominal ultrasound until next week." D. "I will have the health care provider change the order to a transabdominal ultrasound." E. "Because you are in your first trimester, the uterus and embryo are difficult to visualize with a transabdominal ultrasound."

CORRECT B. "If it makes you feel more comfortable, you can insert the probe yourself." Patients may feel more comfortable inserting the probe themselves for a transvaginal ultrasound. E. "Because you are in your first trimester, the uterus and embryo are difficult to visualize with a transabdominal ultrasound." The uterus, ovaries, and embryo are deep within the pelvis during the first trimester; therefore, transvaginal ultrasound is often used.

A patient with a 2-year-old child with cystic fibrosis declines genetic counseling during her antepartum visit, stating that the birth defect has "already happened"; therefore, her future children will not be affected. Which is the appropriate nursing education for this patient? A. "The risk of a child developing cystic fibrosis increases with maternal age." B. "This infant has an equal risk of developing cystic fibrosis as your first child." C. "The risk of your infant developing cystic fibrosis is less than your first child, but it is still present." D. "There is no longer a risk for your infant developing cystic fibrosis, but genetic counseling can help detect other abnormalities."

CORRECT B. "This infant has an equal risk of developing cystic fibrosis as your first child." Having one child with a birth defect does not change the risk for developing the same defect in subsequent children. This response clarifies the patient's misconception about genetic factors of cystic fibrosis and is appropriate for the patient.

The nurse obtains the history from the pregnant woman during the initial prenatal visit. Which would concern the nurse? A. A pregnant woman who consumes 400 mcg of folic acid daily. B. A pregnant woman with type I diabetes and who is on insulin. C. A pregnant woman who had a full-term viable pregnancy two years ago. D. A pregnant woman who denies history of sexually transmitted infection.

CORRECT B. A pregnant woman with type I diabetes and who is on insulin. Type I diabetes would concern the nurse as this increases the risk of fetal anomalies, spontaneous abortion, and maternal complications such as preeclampsia.

Which assessments are performed during the initial prenatal visit? Select all that apply A. Glucose tolerance test B. Calculation of due date C. Prenatal lab panel with blood draw D. Identification of potential risk factors E. Evaluation of the condition of the reproductive organs

CORRECT B. Calculation of due date The menstrual history is taken and used to determine the patient's estimated date of delivery based on 40 weeks' gestation. C. Prenatal lab panel with blood draw Prenatal lab panels, including complete blood count (CBC), rapid plasma regain (RPR), hemoglobin and hematocrit (H&H), human immunodeficiency virus (HIV), hepatitis B (Hep B), and Rubella, are drawn at the initial prenatal visit. D. Identification of potential risk factors Risk factors are determined from a thorough history conducted at the initial prenatal visit. E. Evaluation of the condition of the reproductive organs A bimanual pelvic exam is performed on initial visit to assess the internal genitalia including the uterus and ovaries.

Which actions by the nurse are appropriate when speaking with a patient who needs fetal diagnostic testing? Select all that apply. A. Remind the patient that an abnormal result almost certainly indicates a fetal abnormality. B. Explain how repeated testing benefits the patient and fetus even though it may seem tedious. C. Provide the patient with a clear explanation of why the test is indicated and what abnormalities it can detect. D. Help the patient understand that there is a baseline risk for fetal abnormalities even when all results are normal. E. Being honest with the patient if the nurse disagrees with the patient's decision to continue or terminate the pregnancy.

CORRECT B. Explain how repeated testing benefits the patient and fetus even though it may seem tedious. Explaining the benefit of repeated testing helps the patient understand how the tests benefit both the patient and the infant. This helps to address potential frustration about repeated testing and is an appropriate action for the nurse to take. C. Provide the patient with a clear explanation of why the test is indicated and what abnormalities it can detect. Providing the patient with a clear explanation of why the test is indicated and what abnormalities it can detect helps the patient understand the purpose and potential outcomes of the test. Providing the patient with this information is an appropriate action by the nurse. D. Help the patient understand that there is a baseline risk for fetal abnormalities even when all results are normal. Helping the patient to understand that a baseline risk exists even if all test results are normal helps the patient understand that normal test results do not guarantee the birth of a perfect infant. This helps the patient set realistic goals and is an appropriate action for the nurse to take.

Why should patient education focus on smoking cessation during pregnancy? A. Risk of low blood sugar B. Risk of premature birth C. Risk of congenital anomalies D. Risk of increased fetal circulation

CORRECT B. Risk of premature birth Smoking during pregnancy is related to preterm labor and low birth weight, both of which are associated with other neonate complications. It is crucial for patients to be educated about the potential risks of smoking

A pregnant patient should expect which of the following information to be included during a childbirth preparation class? Select all that apply A. Newborn care B. Role of support person C. Comfort relief during labor D. The labor and birth process E. Proper positioning and correct latch for lactation

CORRECT B. Role of support person The role of the support person would be included in a childbirth preparation class. This helps inform the support person how to assist the mother during labor and birth. Increased knowledge will decrease anxiety for the support person as well. C. Comfort relief during labor Comfort relief during labor would be included in a childbirth preparation class. This would increase the woman's knowledge of different comfort relief measures to help her cope with the pain of labor and birth. D. The labor and birth process The labor and birth process should be included in a childbirth preparation class. This information helps pregnant women understand what to expect during labor and birth. It will help decrease anxiety related to fear of the unknown as well.

A patient with a family history of spina bifida is 16 weeks pregnant. Which is the priority action for the nurse to take? A. Obtain an order for RhoGAM. B. Schedule the patient for amniocentesis. C. Refer the patient to genetic counseling. D. Schedule the patient for a transabdominal ultrasound.

CORRECT B. Schedule the patient for amniocentesis. Amniocentesis should be scheduled between 15 and 20 weeks when there is adequate amniotic fluid present to test AFP level, which is used in screening for neural tube defects such as spina bifida. Scheduling this test should be a priority action for the nurse to take.

Which behaviors should the nurse instruct the woman to avoid during pregnancy? Select all that apply. A. Exercise. B. Smoke marijuana to aid in sleep. C. Consume 8-10 glasses of water per day. D. Drink one glass of white wine per day for heart health. E. Drink 5-6 cups of coffee per day to avoid caffeine withdrawal.

CORRECT B. Smoke marijuana to aid in sleep. Taking any illicit drugs during pregnancy should be discouraged, regardless of the reason. Marijuana is associated with neurobehavioral defects in newborns D. Drink one glass of white wine per day for heart health. Drinking alcohol during pregnancy should be discouraged because it can cause serious birth defects. Even small amounts of alcohol are not recommended. E. Drink 5-6 cups of coffee per day to avoid caffeine withdrawal. Consuming large amounts of caffeine during pregnancy should be discouraged, regardless of the reason. It can cause spontaneous abortion and preterm labor.

Which measures can a pregnant woman take to prevent or relieve heartburn? Select all that apply A. Drink coffee B. Quit smoking C. Avoid fried chicken D. Eat a snack just before bed E. Take a brief walk after eating F. Eat 5-6 small meals throughout the day

CORRECT B.Quit smoking Smoking increases heartburn and should be avoided completely. C. Avoid fried chicken Fried foods increase heartburn and should be avoided. E. Take a brief walk after eating Sitting upright (or walking) for at least one hour after eating promotes gastric emptying and decreases heartburn. F. Eat 5-6 small meals throughout the day Eating small, frequent meals allows for gastric emptying and decreases heartburn.

Which statement made by a patient having an amniocentesis indicates an understanding of the risks associated with the diagnostic test? A. "I will not need RhoGAM after this even though I am Rh-negative." B. "There is a moderate risk I will get an infection as a result of this test." C. "Because the needle is guided by ultrasound, the risk of injury to the baby is small." D. "Because I am 14 weeks gestation, there is a higher chance of the baby having a foot deformity after the procedure."

CORRECT C. "Because the needle is guided by ultrasound, the risk of injury to the baby is small." Risk of injury to the fetus or umbilical cord is low when ultrasound is used to guide needle insertion. This statement indicates that the patient understands the risks of amniocentesis.

Question 3 of 6 A woman comes in for her first prenatal visit. The woman tells the nurse that this is her fourth pregnancy. She had a miscarriage with her first pregnancy at five weeks. She had a 39-week viable male with her second pregnancy. She had 35-week twins with her third pregnancy, and they are all living. How would the nurse record her obstetric history using the GTPAL system? A. G4T2P1A1L3 B. G4T1P2A1L3 C. G4T1P1A1L3 D. G4T1P2A1L2

CORRECT C. G4T1P1A1L3 G4T1P1A1L3 is correct because there is one term pregnancy, one preterm pregnancy, one abortion, and three living children according to the woman's OB history.

A pregnant woman reports constipation. In which ways can the woman prevent or relieve constipation? Select all that apply. A. Drink caffeinated coffee in the mornings B. Increase consumption of dairy products C. Take over-the-counter docustate sodium D. Consume at least eight glasses of water a day E. Walk or engage in light aerobic exercise daily F. Eat green leafy vegetables, whole grains, and fruit

CORRECT C. Take over-the-counter docustate sodium Stool softeners such as docusate sodium increase water content of the stool and promote bowel movement, thus relieving constipation. D. Consume at least eight glasses of water a day Increasing fluid intake will increase peristalsis, soften the stool, and decrease constipation. E. Walk or engage in light aerobic exercise daily Walking and light aerobic exercise will increase peristalsis and decrease constipation. F. Eat green leafy vegetables, whole grains, and fruit Eating fiber-enriched foods helps to prevent constipation.

On assessment, the nurse learns that a male toddler born with a congenital heart defect lives with his parents and two young siblings in the Midwest. The mother is pregnant with a fourth child. Which additional information would the nurse need to know to evaluate the risk for the fourth child being born with a heart defect? Select all that apply A. Socioeconomic status of the family B. The mother's family history of tobacco use C. Variations of symptoms due to time or season D. Sex of family members born with a heart defect E. Number of close relatives born with a heart defect

CORRECT C. Variations of symptoms due to time or season Seasonal variations of symptoms are a factor that may influence the risk of developing a heart defect. The nurse would need this information in order to evaluate the fourth child's risk for developing a heart defect. D. Sex of family members born with a heart defect Sex of affected family members is a factor that may influence the risk of developing a heart defect, as it can help determine whether the trait is X-linked. The nurse would need this information in order to evaluate the fourth child's risk for developing a heart defect. E. Number of close relatives born with a heart defect Number of affected relatives is a factor that may influence the risk of developing a heart defect. The nurse would need this information in order to evaluate the fourth child's risk for developing a heart defect.

Which question should the antepartum nurse ask patients when determining the need for genetic counseling? A. "Have you ever had a miscarriage?" B. "Have you ever been diagnosed with depression or anxiety?" C. "Does anyone in your family need to use a wheelchair or a cane?" D. "Do you have any relatives who have learning problems or are developmentally delayed?"

CORRECT D. "Do you have any relatives who have learning problems or are developmentally delayed?" Family history of developmental disabilities is an indicator for genetic counseling, as it can indicate a patient at increased risk for fetal abnormalities.

The antepartum nurse is caring for a patient who has a 2-year-old child with Down syndrome. The patient declines genetic counseling for her current pregnancy because she states, "there is nothing I can do about it now." Which education is appropriate for this patient regarding the use of genetic counseling? A. "If genetic counseling is done early in the pregnancy, defects can be prevented." B. "Genetic counseling is necessary because the risk for Down syndrome increases with each child." C. "Genetic counseling can help us determine if the abnormality came from you or your partner." D. "Genetic counseling can help us determine the risk of your second child being born with Down syndrome."

CORRECT D. "Genetic counseling can help us determine the risk of your second child being born with Down syndrome." Genetic counseling is used to determine the risk for anomalies in subsequent pregnancies after the first child is born.

The nurse is caring for a patient who has just received genetic counseling and is visibly distressed. The patient tearfully explains to the nurse that the counselor informed her that there was a 50% chance of her child having cystic fibrosis and asks, "What should I do about my pregnancy?" Which response from the nurse is appropriate for this patient? A. "Did the counselor say if there was anything you could do to lower the chance?" B. "We can get you information about adoption, if you think you might want to look at it." C. "A 50% chance isn't very high. There's an equal chance your baby will be perfectly healthy." D. "I know you're faced with a difficult decision, but I'm here to support the choices you make."

CORRECT D. "I know you're faced with a difficult decision, but I'm here to support the choices you make." Genetic counseling is nondirective. The patient's autonomy should be respected, even when the patient does not know what to do. This response is appropriate because it acknowledges the patient's distress but does not make a decision for the patient.

Which patient statement would signal a contraindication for a transcervical chorionic villus sampling (CVS) procedure? A. "My doctor told me I have preeclampsia." B. "My last ultrasound said I'm 10 weeks pregnant." C. "I had a baby before this one, but it had neurological defects." D. "I took fluconazole (Diflucan) yesterday for a yeast infection."

CORRECT D. "I took fluconazole (Diflucan) yesterday for a yeast infection." Vaginal infection is a contraindication for transcervical CVS due to increased risk of uterine infection. The patient would require transabdominal CVS instead.

When scheduling a diagnostic procedure, a patient tells the nurse that she plans to terminate the pregnancy if an abnormality is found. Which is an appropriate response from the nurse? A. "What made you come to that decision?" B. "That isn't what I would do in your situation, but the decision is yours to make." C. "I will give you contact information for support groups for parents of children with disabilities in case you change your mind." D. "I will give you printed information on how to schedule pregnancy termination so you know what to do if the test finds an abnormality."

CORRECT D. "I will give you printed information on how to schedule pregnancy termination so you know what to do if the test finds an abnormality." This response supports the patient's decision and provides follow-up care to assist the patient.

A patient whose ultrasound resulted in normal findings expresses relief that the fetus is "completely healthy." Which response from the nurse is appropriate? A. "Follow-up ultrasound scans will be necessary but only to measure fetal growth." B. "It can take more than one ultrasound to determine if your baby is completely healthy." C. "I'm happy that this test can give you peace of mind; most ultrasounds come back normal." D. "I'm very happy that you feel relieved; however, additional tests may still be indicated to detect other possible abnormalities."

CORRECT D. "I'm very happy that you feel relieved; however, additional tests may still be indicated to detect other possible abnormalities." Ultrasound cannot detect every structural abnormality, and it cannot detect abnormalities that do not affect the structure of the fetus. This response is appropriate because it addresses the patient's misconception regarding ultrasound testing.

How can the pregnant woman relieve symptoms of nausea and vomiting that occur in the first trimester upon awakening in the morning? A. Drinking a cup of coffee in the morning B. Taking prenatal vitamins in the morning C. Consuming lots of fluids with solid foods D. Eating a dry piece of toast in the morning

CORRECT D. Eating a dry piece of toast in the morning Eating dry carbohydrates before rising in the morning can help absorb gastric acid and therefore decrease nausea and vomiting.

Which pregnancy discomfort requires immediate follow-up with a health care provider? A. Backache B. Constipation C. Hemorrhoids D. Epigastric pain

CORRECT D. Epigastric pain Epigastric pain can indicate elevated blood pressure/preeclampsia.

Which condition is suspected by a urine dipstick screen that shows 2+ glucosuria? A. Dehydration B. Preeclampsia C. Preterm labor D. Gestational diabetes

CORRECT D. Gestational diabetes Gestational diabetes would be a concern if the urine dipstick shows 2+ glycosuria.

The nurse should instruct the patient to consume which food or drink to enhance the absorption of iron during pregnancy? A. Milk B. Water C. Cheese D. Orange juice

CORRECT D. Orange juice Vitamin-C-enriched fluids/foods, such as orange juice, enhance the absorption of iron.

Which type of exercise can help with lower back discomfort during pregnancy? A. Kegel B. Sit-ups C. Push-ups D. Pelvic tilts

CORRECT D. Pelvic tilts Pelvic tilt exercises strengthen the muscles of the lower back and decrease backache during pregnancy.

A pregnant patient is traveling for a family vacation. She will be driving in the car for eight hours. What should the nurse instruct the patient to do while traveling? A. Decrease fluid intake. B. Drive with minimal stops. C. Avoid travel by automobile. D. Stop frequently to walk around.

CORRECT D. Stop frequently to walk around. The nurse should instruct the patient to stop every two hours and ambulate for ten minutes. This will help prevent the formation of clots in the lower extremities.

A woman at eight weeks gestation asks the nurse why she is so nauseated with her pregnancy. What should the nurse explain to the woman? A. Nausea is related to a decrease in progesterone. B. Nausea is caused by consumption of excess sugar. C. Nausea is not aggravated by foul odors and fatigue. D. The exact cause of nausea during pregnancy is unknown. It may be related to an increase in hCG.

CORRECT D. The exact cause of nausea during pregnancy is unknown. It may be related to an increase in hCG. Nausea may be related to an increase in hCG.

What contributes to helping an infant to fight infection within the first year of life? (SELECT ALL THAT APPLY) Breast milk Vaccinations Fully functional respiratory system The mother's transplacental antibodies Bottle-fed cow's milk supplemented with Vitamins A, C, and D

CORRECT- Breast milk The breast milk contains IgA and proteins including lactoferrin, lysozyme, and serum albumin. The infant benefits from maternal IgA since the baby's immune system can only make 20% of the adult level. Vaccinations Vaccinations for diseases such as rotavirus and hepatitis B are given to infants, which helps them fight infection .The mother's transplacental antibodies Transplacental antibodies help supplement the infant's immune system until the infant is around 4 months of age. IgG is the most significant transplacental antibody. INCORRECT: Fully functional respiratory system The infant's respiratory system is not fully functional but rather immature along with the immune system, putting the infant at risk for infection. Bottle-fed cow's milk supplemented with Vitamins A, C, and D Cow's milk contains too little iron, and its high renal solute load and unmodified derivatives can put small infants at risk for dehydration.

Which activities correspond to normal motor development during the first year of life? (SELECT ALL THAT APPLY) 10-12 months: can stand alone 8-9 months: sits steadily, unsupported 1-2 months: can roll from front to back 4-5 months: turns from back to abdomen 6-7 months: sits and leans forward on both hands

CORRECT: 10-12 months: can stand alone At 10-12 months the child has the motor development to be able to stand on his or her own. 8-9 months: sits steadily, unsupported The baby's motor development will be mature enough to allow the child to sit steadily, unsupported. 6-7 months: sits and leans forward on both hands At 6-7 months the baby's motor development will be mature enough to sit and lean forward on both hands (tripod sitting). INCORRECT: 1-2 months: can roll from front to back At 1-2 months the child will not have enough motor development to roll front to back. 4-5 months: turns from back to abdomen At 4-5 months children usually cannot turn from back to abdomen on their own.

Which statements describe the cognitive difference in regard to object permanence between a 6-month-old and a 9-month-old? Select all that apply. A 6-month-old will not notice if you hide a toy. A 9-month-old will look for a toy if it is hidden from view. A 6-month-old will look for a toy if it is hidden from view. A 6-month-old will look for the toy and will likely show no interest once the toy is found. A 9-month-old will ignore the toy if it is hidden and immediately seek another object of interest.

CORRECT: A 6-month-old will not notice if you hide a toy. At 6 months the cognitive development of the child will not be mature enough to understand that the object still exists if it is hidden from view. A 9-month-old will look for a toy if it is hidden from view. The development of object permanence occurs in children around the age of nine months and therefore a child at this age will look for a toy if it is hidden from view. INORRECT: A 6-month-old will look for a toy if it is hidden from view. At six months, the child is more likely to show interest with the toy in full view. A 6-month-old will look for the toy and will likely show no interest once the toy is found. This behavior does not describe the concept of object permanence and is not the typical behavior of a six month old. A 9-month-old will ignore the toy if it is hidden and immediately seek another object of interest. This statement does not describe the phenomenon of object permanence found in a typical 9-month-old. The child will likely look for the toy since they now understand that the object still exists even though it is not in view.

Which can influence the development of both speech and language in infants? Select all that apply. Anatomical defects Genetics/family history Sociodemographic factors Lack of access to television and computers Quality of communication in the environment

CORRECT: Anatomical defects This can affect speech and language development. Children with cleft lip and palate for example may have difficulty reaching normal language milestones. Genetics/family history This factor can have an effect on speech and language development. Genetics and family histories will always be a consideration when evaluating a child's speech and language milestones. Sociodemographic factors Sociodemographic factors can have an effect on both speech and language development. Quality of communication in the environment This can affect speech and language development in infants, such as in a home where family members place a high value on education, with family members who are bilingual, or if the child has other siblings to converse with. INCORRECT: Lack of access to television and computers Avoiding TV and computers should not influence speech and language development in infants.

Which sentence best describes a healthy parent-infant attachment during the first month? Bonding is critical for normal development and survival. Bonding is critical but too much love can create a needy child. Bonding is important but meeting every emotional need will spoil the child. Bonding is necessary but the infant has built-in mechanisms to help if the child's needs are not met.

CORRECT: Bonding is critical for normal development and survival. This sentence best describes the relationship that needs to exist for normal development and survival in the infant during the first few months of life. INCORRECT: Bonding is critical but too much love can create a needy child. This statement does not describe a healthy parent-infant attachment in the first few months of life. A child's emotional development will be nurtured with a lot of love and this will not lead to neediness. Bonding is important but meeting every emotional need will spoil the child. Bonding is important and is critical for the child's survival. This type of attachment will not spoil the child but instead reinforce a healthy infant-maternal relationship. Bonding is necessary but the infant has built-in mechanisms to help if the child's needs are not met. This sentence does not describe a healthy parent-infant attachment during the first months of life. The infant does not have a built-in mechanism to survive without the care of the parent.

Which situation best describes a 6-month-old's ability to maintain a standing position? The 6-month-old will kick slightly to avoid bearing weight. The 6-month-old will stand on his or her own and bear full weight. The 6-month-old can bear significant weight when held in a standing position. The 6-month-old's knees will bend and they will bear no significant weight.

CORRECT: The 6-month-old can bear significant weight when held in a standing position. At 6 months the baby's muscles are developing and have reached a point where they are strong enough to allow the baby to bear significant weight when held in the standing position. INCORRECT: The 6-month-old will kick slightly to avoid bearing weight. This scenario does not reflect the ability of a 6-month-old to maintain a standing position. The 6-month-old will stand on his or her own and bear full weight. This description best fits the motor strength of a child who is 12 months old. The 6-month-old's knees will bend and they will bear no significant weight. This situation does not describe a 6-month-old's ability to maintain a standing position. This would describe the motor strength of a 2-month-old.

Which aspect of ear development leads to increased susceptibility to ear infections during the first two year of life? The eustachian tube is short and horizontal. The external acoustic meatus is short and vertical. The auricle is narrow and can block the eustachian tube. The outer tympanic membrane is porous and not highly vascularized.

CORRECT: The eustachian tube is short and horizontal. The eustachian tube is short and horizontal in children, making them more prone to infection. The eustachian tube connects to the middle ear and provides air and drains fluid, which helps to fight infection. INCORRECT: The external acoustic meatus is short and vertical. The external acoustic meatus is not short and vertical but rather elongated and horizontal. Also, the middle ear is more prone to infection and the acoustic meatus is part of the outer ear. The auricle is narrow and can block the eustachian tube. The auricle does not block the eustachian tube since the auricle is part of the outer ear and the eustachian tube attaches to the middle ear. The outer tympanic membrane is porous and not highly vascularized. The tympanic membrane is not porous and it is vascularized. Also the tympanic membrane is the barrier between the outer and middle ear. The outer ear, which includes the pinna and the ear canal, is not as prone to infection as the middle ear in children.

Which statement reflects the consequence of a mother suffering from postpartum depression in regard to an infant's psychosocial development? The child will not be affected by the mother's depression. The infant will develop empathy and respond to the mother's sadness. The infant will develop a preference for other caregivers and may avoid eye contact until two years of age. The infant will struggle to develop the basic sense of trust that the infant's needs will be met and instead develop a sense of suspicion or mistrust.

CORRECT: The infant will struggle to develop the basic sense of trust that the infant's needs will be met and instead develop a sense of suspicion or mistrust. The child needs to develop a sense of trust with the mother and the mother's depression will interfere in the positive reciprocal process that enables psychosocial development in the infant. INCORRECT: The child will not be affected by the mother's depression. This statement does not reflect the consequence of a mother suffering from postpartum depression in regard to the infant's psychosocial development. The child may be affected by the mother's depression. The infant will develop empathy and respond to the mother's sadness. This statement does not reflect the consequence of a mother suffering from postpartum depression in regard to the infant's psychosocial development. The child is too young to develop empathy and will therefore not be able to understand the mother's inability to connect with the child. The infant will develop a preference for other caregivers and may avoid eye contact until two years of age. This statement does not reflect the consequence of a mother suffering from postpartum depression in regard to the infant's psychosocial development. The child will still prefer the mother but the bond will not develop appropriately.

Which is a common physical growth pattern seen in infancy during the first to second month? Upper limbs and head grow faster. Lower limbs and GI tract develop faster. Cardiovascular system and lower limbs develop faster. Respiratory and immune systems develop at a rapid pace.

CORRECT: Upper limbs and head grow faster. A faster growth of the upper limbs and head is a common pattern seen in the first two months of life. Head circumference measurements are taken at each health provider visit to ensure the infant is showing normal growth patterns. INCORRECT: Lower limbs and GI tract develop faster. A faster pace is not seen in the lowers limbs and GI tract. The GI tract GI develops relatively slowly, which is why such care needs to be taken in feeding infants. Cardiovascular system and lower limbs develop faster. While the cardiovascular system develops in the fetus at a faster pace, a rapid pace of growth is not seen in infancy during the first two months. Respiratory and immune systems develop at a rapid pace. A rapid pace of growth is not seen in the respiratory and immune systems. For this reason, infants are more vulnerable and susceptible to infection.

Why is an understanding of motor development important for keeping a child safe when an infant's mobility increases around 10 months of age? Select all that apply. The child has no understanding of consequences. The child has a clear understanding of boundaries. The child now has an increased ability to move about. The child now has a naturally increased level of curiosity. The child now has an increased ability to reach for objects.

CORRECT: The child has no understanding of consequences. An increased level of curiosity without the cognitive ability to understand consequences puts the infant at great risk for accidents and injury. The child now has an increased ability to move about. An increased ability to move about, reach objects, and explore their world places infants at great risk for accidents and injury. The child now has a naturally increased level of curiosity. Now that they child can grab at objects, heavy containers will become a hazard. An increased ability to move about, reach objects, and explore their world places infants at greater risk for accidents and injury. The child now has an increased ability to reach for objects. They now have the ability to grasp things and an increased ability to move about, reach objects, and explore their world, placing infants at greater risk for accidents and injury. INCORRECT: The child has a clear understanding of boundaries. The child does not have a clear understanding of boundaries at this age, which will need to be kept in mind when the child's safety is considered.

A woman at 37 weeks' gestation calls and reports, "my water broke and I have bloody show. I am changing my pad every ten minutes." Which does the nurse correctly advise the woman to do? Engage in activity such as walking Call her health care provider and go to the hospital or birthing center Advise the woman that overreacting is not good for her or the baby and to call back when she is in "true labor" Remain home, and only come in to the hospital or birthing center if contractions are regular, frequent, and intense

Call her health care provider and go to the hospital or birthing center Heavy bleeding and/or a gush of fluid from the vagina should be evaluated by a health care provider at the hospital or birthing center.

A nurse completes an assessment of a 28 weeks' neonate. The findings include an axillary temperature of 89.6 degrees Fahrenheit, listlessness, and a moderate emesis of digested breastmilk. Which is a priority action for the nurse to take? Assess for gastric residuals. Assess the abdominal girth for distension. Call the health care provider with the assessment findings. Observe the neonate for tachypnea, grunting, and costal retractions.

Call the health care provider with the assessment findings.

What is the single most important indicator of the progress of labor?

Cervical Dilation

Which labor assessment finding best distinguishes between true and false labor? Cervical effacement and dilatation Mild discomfort felt in the abdomen and pelvis Contractions that subside with increased activity No significant change in cervical effacement or dilatation

Cervical effacement and dilatation The best distinction between the two is that the contractions of true labor cause progressive changes in the cervix. Effacement and dilatation occur with true labor contractions.

The nurse is caring for a breastfed newborn with yellow sclera and lethargy. Which priority diagnostic test would the nurse anticipate being ordered? Obtain chest X-ray Check a bilirubin level Assess the child's blood glucose Determine white blood cell count

Check a bilirubin level Icteric sclera and lethargy are signs of jaundice. The nurse should obtain a bilirubin level.

Nursing Assessments During Third Stage

Check vitals every 15 minutes Check for signs of placental separation from uterus Assignment of 1 and 5 minute Apgar scores

A newborn born to a mother dependent on opiates is irritable, diaphoretic, and difficult to console. Which nursing interventions are priorities? Select all that apply. Cluster nursing care activities. Rock the baby during feedings. Provide the newborn a pacifier. Keep bright lights on in the nursery. Swaddle the infant in a flexed position.

Cluster nursing care activities. Provide the newborn a pacifier. Swaddle the infant in a flexed position.

Which characteristics of contractions are indicative of true labor in a primigravida? Select all that apply. Lightening Contractions remain consistent in intensity Contractions begin in abdomen, but spread Consistent contractions increasing in duration Progressive effacement and dilatation of the cervix

Consistent contractions increasing in duration Contractions occurring in a consistent pattern of increasing duration indicate true labor. Progressive effacement and dilatation of the cervix There is progressive effacement and dilatation of the cervix during true labor.

The nurse is caring for a patient in labor and has identified a severe deceleration in fetal heart rate. The health care provider has subsequently diagnosed a cord prolapse. Which actions should the nurse perform? Select all that apply. Continue assessing fetal heart rate. Assess the patient's level of anxiety. Ensure that the patient's hips are elevated above the head. Palpate the prolapsed umbilical cord to verify presence of pulse. Evaluate the family's understanding of the need for immediate delivery.

Continue assessing fetal heart rate. Assessing fetal heart rate allows the nurse to ensure that pressure on the cord is minimized. Assess the patient's level of anxiety. The nurse should assess the patient's anxiety and provide education to address the patient's concerns. Ensure that the patient's hips are elevated above the head. Ensuring that the patient's hips are elevated above the head helps maintain relief of cord compression and ensure fetal oxygenation. Evaluate the family's understanding of the need for immediate delivery. The nurse should include the family in education as much as possible and evaluate the family's understanding of the patient's condition.

Nursing Interventions During Second Stage of Labor

Continue to monitor fetus and client Assist in positioning for effective pushing Assist in partner involvement and encourage bearing down efforts Promote rest between contractions Provide comfort measures such as cold compress Cleanse client's perineum if feces is expelled during pushing Prepare for episiotomy if needed Provide feedback on labor progress Prepare for care of neonate -check oxygen flow and tank on warmer -preheat radiant warmer -lay out newborn stethoscope and bulb syringe -Have resuscitation equipment in working order and emergency equipment available -check suction apparatus

A mother with a history anorexia voices concern over her ability to nourish her newborn by breastfeeding. Which information should the nurse include in patient teaching? Select all that apply. Formula supplementation will be required. Anorexic mothers are unable to breastfeed. Continue to take a multivitamin while breastfeeding. Ensure a balanced diet to maintain energy levels. Breast milk of undernourished mothers contains nearly the same nutrients.

Continue to take a multivitamin while breastfeeding. All breastfeeding mothers should continue to take a multivitamin during breastfeeding to prevent vitamin depletion. Ensure a balanced diet to maintain energy levels. A balanced diet will help maintain the mother's energy while breastfeeding. Breast milk of undernourished mothers contains nearly the same nutrients. The mother's body prepares breast milk to contain all necessary nutrients for the newborn, even if the mother is undernourished.

Match the age with the critical milestone in language development. Cooing with a reciprocal smile Babbling is common. Repeating sounds to self Increase interest in sounds 1 year 1-3 months 4-6 months 3-4 months 6-8 months 9 months

Cooing with a reciprocal smile 1-3 months Babbling is common. 3-4 months Repeating sounds to self 4-6 months Increase interest in sounds 6-8 months

A pregnant patient with a history of opioid abuse reports that she has cut down on her opioids but that she is having difficulty abstaining and is feeling depressed and alone. Which action by the nurse is necessary to provide appropriate ongoing care for the patient and her fetus? Administer butorphanol intravenously. Coordinate therapy with mental health care provider. Send patient to Emergency Department for immediate evaluation. Praise her reduction in use of opioids and provide reassurance that she is doing well on her own.

Coordinate therapy with mental health care provider.

A 2-day-old neonate has a serum bilirubin level of 18.9 mg/dL. Double bank phototherapy is ordered. Which is the most important intervention for the nurse when caring for the neonate? Placing the neonate naked under the phototherapy Supplementing with formula after each breastfeeding Weighing the newborn before and after every breastfeeding Covering the eyes with an eye mask when under phototherapy lights

Covering the eyes with an eye mask when under phototherapy lights

A 38-year-old nulliparous woman bearing twins asks the nurse why she is at risk for PROM. Which statement by the nurse is appropriate? A. "Gaining weight progressively predisposes a woman to PROM." B. "Since this is your first pregnancy, you have an increased risk for PROM." C. "Being pregnant at an advanced maternal age could precipitate PROM." D. "Bearing multiple fetuses in one pregnancy can lead to overdistention of the uterus, which could result in PROM."

D. "Bearing multiple fetuses in one pregnancy can lead to overdistention of the uterus, which could result in PROM."

A woman is admitted to the Labor and Delivery unit with premature rupture of membranes at 31 weeks' gestation and is experiencing symptoms of preterm labor. The nurse has assessed the woman for signs of infection and is administering corticosteroids following an order from the health care provider. Which statement should the nurse make to explain why corticosteroids have been ordered? A. "Corticosteroids are administered to help delay labor." B. "Corticosteroids are administered to help "quiet" uterine activity." C. "Corticosteroids are administered to help fight infection in the mother." D. "Corticosteroids are administered to help speed your baby's lung maturity."

D. "Corticosteroids are administered to help speed your baby's lung maturity."

A patient in labor is in visible distress and reports fear of pushing too hard and "tearing something." Which response from the nurse is appropriate? A. "Cervical ripening may help ensure the tissue does not tear." B. "If you feel tearing sensations, you should not push any harder." C. "If you feel like you might tear something, I may need to request the health care provider schedule you for cesarean delivery." D. "The team can't guarantee that you won't have some tearing, but your body will stretch to accommodate the baby."

D. "The team can't guarantee that you won't have some tearing, but your body will stretch to accommodate the baby."

A patient with HIV asks whether skin-to-skin contact during breastfeeding will harm her newborn. Which is the best response from the nurse? A. "Breastfeeding is always best." B. "You can breastfeed or bottle feed; it is up to you." C. "You should bottle feed until your breast milk comes in." D. "You should bottle feed formula to reduce the risk of transmission of HIV to your baby."

D. "You should bottle feed formula to reduce the risk of transmission of HIV to your baby." Bottle feeding formula is the recommended method of feeding for mothers with HIV as breastfeeding increases the risk of HIV transmission.

A nurse in L&D is planning care for a newly admitted client who reports she is in labor and has been having vaginal bleeding for 2 weeks. Which of the following should the nurse indicate in the plan of care? A. Inspect the introitus for prolapsed cord B. Perform a test to identify ferning pattern C. Monitor station of presenting part D. Defer vaginal exams

D. Defer vaginal exams rationale: should not be performed until placenta privia and placenta abruptio has been ruled out as cause for bleeding

The fetus of a patient who presents with anemia accompanied by hyperbilirubinemia is at risk for which complication during pregnancy? A. Eclampsia B. Placenta previa C. Gestational diabetes D. Fetal growth restriction

D. Fetal growth restriction hyperbilirubinemia is suggestive of sickle cell anemia, which is accompanied by a risk of fetal growth restriction if not controlled.

Which nursing intervention is priority for a pregnant patient with cardiovascular disease? A. Reduce iron-rich foods B. Avoid immunizations for flu and pneumonia C. Recommend a weight gain of at least 35 lbs with pregnancy D. Monitor for changes in blood pressure, pulse, and respiratory rate

D. Monitor for changes in blood pressure, pulse, and respiratory rate Monitoring for changes in vital signs is a priority in nursing care for a pregnant patient with cardiovascular disease. These changes can provide an early indication of changes in cardiac function.

An HIV-positive patient is 36-weeks pregnant. Which is a management strategy to reduce the risk of transmission of the infection to the fetus? A. Early induction of labor. B. Administration of penicillin. C. Administration of intravenous zidovudine just prior to vaginal delivery. D. Perform a scheduled Cesarean delivery at 38 weeks gestation.

D. Perform a scheduled Cesarean delivery at 38 weeks gestation. Performing a scheduled Cesarean delivery at 38 weeks' gestation is one management strategy used for reducing the risk of transmission to the fetus. Even with this option, antiretroviral therapy (ART) is still recommended throughout the pregnancy.

A woman at 34 weeks' gestation reports having four contractions in one hour and menstrual-like abdominal cramps. Which condition is the woman experiencing? A. Stillbirth B.Miscarriage C. Chorioamnionitis D. Preterm labor (PTL)

D. Preterm labor (PTL)

The nurse is caring for a patient experiencing prolonged labor. The patient reports intense back pain that radiates down to the leg. Which is the appropriate nursing intervention for this patient? A. Elevate the head of the bed. B. Notify the health care provider. C. Administer the ordered pain medication. D. Reposition the patient onto her hands and knees.

D. Reposition the patient onto her hands and knees.

The nurse understands that which is a newborn consequence of group beta strep (GBS) infection? A. Encephalitis B. Microcephaly C. Severe fetal anemia D. Respiratory problems

D. Respiratory problems Pneumonia is a newborn consequence of group beta strep infection.

The nurse includes which information in the patient teaching about viral infections during pregnancy? A. Viral infections occur rarely in pregnancy. B. Viral infections may be treated with antibiotics. C. Viral infections always cause symptoms in the mother. D. Viral infections can result in serious consequences for fetus and newborn.

D. Viral infections can result in serious consequences for fetus and newborn. It is important to include that viral infections can result in serious consequences for the fetus and newborn.

A nurse is caring for a client in the first stage of labor and encourages the client to void every 2 hr. The nurse explains that a... A. full bladder increases the risk for fetal trauma B. full bladder increases the risk for bladder infections C. distended bladder will be traumatized by frequent pelvic exams D. distended bladder reduces pelvic space needed for birth

D. distended bladder reduces pelvic space needed for birth

The nurse is caring for a pregnant patient receiving intravenous (IV) oxytocin for induction of labor. The most recent fetal heart rates indicate fetal bradycardia unresponsive to repositioning. Which order would the nurse anticipate? Discontinue the oxytocin infusion Increase the rate of the oxytocin infusion Administer 5% dextrose with the oxytocin infusion Administer supplementary oxygen by simple face mask

Discontinue the oxytocin infusion Fetal bradycardia (fetal heart rate <110 beats/min lasting for more than 10 minutes) is a sign of potential uterine tachysystole and reduced placental exchange caused by oxytocin administration. Therefore, discontinuing the IV oxytocin infusion is the next step taken to relax the uterus, thus increasing blood flow to the fetus.

The nurse is caring for a neonate who is undergoing phototherapy. What specific care should the nurse plan for this infant? Applying mineral oil to the skin to prevent excoriation Covering the infant's head with a cap to minimize heat loss Regulating radiant heat to maintain optimum skin temperature Discontinuing therapy during feeding to meet the infant's emotional needs

Discontinuing therapy during feeding to meet the infant's emotional needs (Discontinuing therapy during feedings is necessary to ensure psychosocial contact. Mineral oil may block light rays from acting on bilirubin deposits; cleansing after each voiding and defecation will prevent skin excoriation. All parts of the body may contain bilirubin deposits and should be exposed to the light. Radiant heaters are not used; a fluorescent light source is used.)

A nurse is assessing a newborn and notes that the newborn has a single palmar crease at the base of the fingers. The nurse knows that this is a soft sign for what and should be reported to the health care provider? Seizure disorder Down Syndrome Opioid withdrawal Metabolic disorder

Down Syndrome A single palmar crease at the base of the fingers is also known as a simian crease, and is a soft marker for Down syndrome.

A 45-year-old newly pregnant woman has Type 1 diabetes, hypertension, and gastroesophageal reflux disease (GERD). On initial prenatal visit, the antepartum nurse offers genetic counseling services which the patient refuses. Which action is priority for the nurse to take?

Educate the patient as to why genetic testing is indicated. This is the priority action because providing information as to why genetic testing is indicated may change the patient's perception of genetic counseling. The patient has many risk factors associated with birth defects and intelligence disorders. Genetic counseling would benefit the patient and fetus.

Which important anticipatory guidance would parents need to learn prior to the development of the infant's pincer grasp (around 9 months of age)? Select all that apply. Electrical outlets should be covered with socket safety panels. Parents should place stove pot handles away from the child's reach. Parents should keep unsafe objects locked out of reach of the infant. The infant's food items should be cut into small, manageable pieces. Parents should place hand protectors on the child to help protect them from injury.

Electrical outlets should be covered with socket safety panels. Six-month-old infants grasp objects with all their fingers in a raking motion, but 9-month-olds use their thumbs and forefingers which allows them to grab potentially harmful things, putting them at risk for electrical injury. Correct Parents should place stove pot handles away from the child's reach. This would be good anticipatory guidance to give a parent with a child who is 9 months old since the child will be reaching for dangerous objects when in the kitchen. Correct Parents should keep unsafe objects locked out of reach of the infant. Infants will need to be watched closely, and since they are growing in both cognitive development and motor development, this ability provides them with a wider view of the world and creates new ways to play, but this may put them in danger. Correct The infant's food items should be cut into small, manageable pieces. Nine-month-olds use their thumbs and forefingers in a fine motor skill called the pincer grasp, which allows them to grab potentially harmful things and place them in their mouth, which creates a choking hazard. INCORRECT: Parents should place hand protectors on the child to help protect them from injury. This is not a realistic suggestion to offer parents and therefore this advice will not be given for anticipatory guidance on the pincer grasp.

What should be included in a plan of care to limit the development of hyperbilirubinemia in the breastfed neonate? Encouraging more frequent breastfeeding during the first 2 days Instituting phototherapy for 30 minutes every 6 hours for 3 days Substituting formula feeding for breastfeeding on the second day Supplementing breastfeeding with glucose water during the first day

Encouraging more frequent breastfeeding during the first 2 days (More frequent breastfeeding stimulates more frequent evacuation of meconium, thereby preventing resorption of bilirubin into the circulatory system. Phototherapy is the treatment for hyperbilirubinemia, and it is maintained continuously; it does not prevent the development of hyperbilirubinemia. It is not necessary to feed the infant formula. Early breastfeeding tends to keep the bilirubin level low by stimulating gastrointestinal activity. Increasing water intake does not limit the development of hyperbilirubinemia, because only small amounts of bilirubin are excreted by the kidneys.)

A mother has brought in her infant for the infant's 6-month well-child checkup. You notice the infant's left eye intermittently not aligning with the right eye and an inability to focus both eyes on the same spot. What is the appropriate response? Select all that apply. Ensure the infant's "red reflex" is normal. Place the infant in isolation as this can be a sign of conjunctivitis. Do nothing, as infants do not gain extraocular eye coordination until 9 months of age. Stress the importance of eye exercises to help prevent worsening of the condition. Notify the child's provider, as extraocular eye alignment should be achieved by 6 months of age.

Ensure the infant's "red reflex" is normal. This is an important part of any pediatric eye exam. Notify the child's provider, as extraocular eye alignment should be achieved by 6 months of age. Infants under 3 to 4 months of age often lack eye coordination and alignment but should achieve proper coordination by age 4 to 6 months. By 3 months, infants should be able to track and reach for objects as their eye-hand coordination begins to develop.

The nurse is caring for a patient in active labor. The patient's cervix is 6 cm dilated and soft, unchanged from the previous assessment. The patient reports feeling exhausted and "stuck." Nursing management should include which intervention? Guide the patient in pursed-lip breathing Administer supplementary oxygen by nasal cannula Establish reliable peripheral intravenous (IV) access Assist the patient into a standing position for as long as she can tolerate it

Establish reliable peripheral intravenous (IV) access Augmentation of labor is indicated in cases of maternal exhaustion and ineffective contractions. Augmentation is performing by administering IV oxytocin. As a result, it is imperative that the patient have reliable IV access.

Which changes in the maternal estrogen to progesterone level signal the onset of labor? Estrogen levels are equal to progesterone levels. Estrogen levels are lower than progesterone levels. Estrogen levels are higher than progesterone levels. There is no significant change in ratio of estrogen to progesterone.

Estrogen levels are higher than progesterone levels. Estrogen levels equal to progesterone prevent contractions to occur.

First Degree Lacerations

Extends through skin of perineum and doesn't involve muscles

Second Degree Lacerations

Extends through skin, muscles into perineum

Fourth Degree Lacerations

Extends through skin, muscles, anal sphincter and anterior rectal wall

Third Degree Lacerations

Extends through skin, muscles, perineum and anal sphincter muscle

A preterm newborn is admitted to the neonatal intensive care unit (NICU). Which concern is most commonly expressed by NICU parents? Fear of handling the infant Delayed ability to bond with the infant Prolonged hospital stay needed by the infant Inability to provide breast milk for the infant

Fear of handling the infant (Because these infants are so tiny and frail, parents most commonly fear handling or touching them; they should be encouraged to do so by the NICU staff. The primary concern is the infant's fragility, not bonding; however, bonding should be encouraged. Although there may be concerns about a long hospital stay, they are not commonly expressed by mothers. The primary concern is the infant's fragility, not breast-feeding. Breasts may be pumped and breast milk given in gavage feedings.)

The newborn's heel stick reveals a glucose reading of 39 mg/dL. Which priority intervention does the nurse implement? Feed the newborn Swaddle the newborn Administer IV fluids Assess newborn reflexes

Feed the newborn A serum glucose of 39 mg/dL is considered hypoglycemia and should be treated with a feeding.

What consequences may result if an infant's needs are ignored by the parent? Feelings of insecurity Emotional resilience An early development of independence Inappropriate survival mechanism development

Feelings of insecurity If the infant's needs are not met, the child will develop feelings of insecurity at an early age, which can continue throughout life, along with a feeling of dissatisfaction.

The nurse is caring for a patient for whom induction is being considered. Which assessment data are necessary to calculate the patient's Bishop score? Select all that apply. Fetal station Fetal heart rate The patient's cervical dilatation The patient's week of gestation The position of the patient's cervix

Fetal station Bishop's score is calculated on the basis of five parameters. Fetal station would consequently need to be identified. The patient's cervical dilatation Bishop's score is calculated on the basis of five parameters. It would be necessary to determine the dilatation of the patient's cervix, measured in centimeters. The position of the patient's cervix Bishop's score is calculated on the basis of five parameters. The position of the patient's cervix would need to be determined.

Signs of Placental Separation from Uterus

Fundus firmly contracting Swift gush of dark blood from introitus Umbilical cord appears to lengthen as placenta descends Vaginal fullness on exam

After the heel stick of a newborn, the nurse anticipates which laboratory finding? Select all that apply. High WBC Low hematocrit High hemoglobin High platelet count Elevated erythrocytes

High WBC Leukocytes are high in the newborn. High hemoglobin Hemoglobin levels are typically high immediately after delivery. Elevated erythrocytes Red blood cells are higher in the newborn.

A nurse is caring for a neonate whose mother received daily methadone treatment during pregnancy. The neonate is irritable, tremulous, and feeds poorly at the breast. The nurse should assess for which other symptoms? Select all that apply. Weight gain of 0.5% from yesterday. High-pitched cry during a diaper change. Aggressive sucking on the hands and fingers. Bottle feeding 15 mL formula at every feeding. Persistent flexion of the upper and lower extremities.

High-pitched cry during a diaper change. Aggressive sucking on the hands and fingers. Persistent flexion of the upper and lower extremities.

A neonate born at 39 weeks' gestation is small for gestational age. Which commonly occurring problem should the nurse anticipate when planning care for this infant? Anemia Hypoglycemia Protein deficiency Calcium deficiency

Hypoglycemia (Hypoglycemia is common in newborns who are small for gestational age because of malnutrition in utero; the nurse can detect this with a blood glucose test and notify the primary healthcare provider. Polycythemia, not anemia, is more likely to occur. Although a protein deficiency may occur, it is not life threatening at this time. Although hypocalcemia may occur, it is not as common as hypoglycemia.)

A small-for-gestational-age (SGA) newborn has just been admitted to the nursery. Nursing assessment reveals a high-pitched cry, jitteriness, and irregular respirations. With which condition are these signs associated? Hypervolemia Hypoglycemia Hypercalcemia Hypothyroidism

Hypoglycemia (SGA infants may exhibit hypoglycemia, especially during the first 2 days of life, because of depleted glycogen stores and inhibited gluconeogenesis. These are not signs of hypervolemia. Hypervolemia is usually the result of excessive intravenous infusion. It is unlikely that a full-term SGA infant will need intravenous supplementation. Hypercalcemia is uncommon in newborns. These signs are unrelated to hypothyroidism; signs of hypothyroidism are difficult to identify in the newborn.)

The nurse assessing a newborn suspects Down syndrome. Which characteristics support this conclusion? Select all that apply. Hypotonia High-pitched cry Rocker-bottom feet Epicanthal eye folds Singe transverse palmar crease

Hypotonia Epicanthal eye folds Singe transverse palmar crease (Hypotonia is typical of newborns with Down syndrome. Their muscle tone is flaccid; they have less control of the head than a healthy newborn does because of their weak muscles. The single crease across the palm of the hand is typical of newborns with Down syndrome. Epicanthal eye folds give the newborn with Down syndrome the typical slant-eyed appearance. A high-pitched cry is characteristic of newborns with brain damage, cerebral irritability (opioid withdrawal), and cerebral edema (hydrocephaly). Rocker-bottom feet are found in newborns with trisomy 18.)

When should vaginal exams during labor be avoided?

In presence of vaginal bleeding Until placenta previa or placenta abruptio is ruled out

Place the steps of neonatal cardiovascular adaptation in the order in which they occur.

Increase in oxygen level Shift in pressure in the heart and lung Closing of umbilical vessels Closing of ductus arteriosus, foramen ovale, ductus venosis After delivery, the partial pressure of oxygen increases, followed by a shift in the pressure in the heart, lungs, and systemic circulation. The umbilical cord vessels are closed off and then the fetal shunts close.

A patient who is receiving oxytocin demonstrates signs of tachycardia. The nurse places the patient in a lateral position and then notifies the health care provider. The nurse then discontinues the oxytocin infusion. Which is the next nursing action indicated for this patient? Elevate the head of the bed Discontinue the primary nonadditive infusion Increase the rate of intravenous fluids as prescribed Call an emergency code and prepare the patient for cesarean delivery

Increase the rate of intravenous fluids as prescribed Increased fluid volume has the potential to simultaneously reduce uterine tone and enhance oxygenation.

A mother of a 10-month-old comes to the nurse concerned that her child has had two upper respiratory infections in one month and asks if something is wrong with her child. How can the nurse respond? To prevent infection the infant will need prophylactic antibiotics. Infants have an immature immune system, which makes them susceptible to infections. An infant's immune system should be developed by 6 months and so the child needs further evaluation. The infant's infections are a sign that the child will develop asthma.

Infants have an immature immune system, which makes them susceptible to infections. Telling the parent that the child has an immature immune system, making them more susceptible to infection, will be the best explanation for the nurse to give the parent. The nurse can explain to the parent that the airways are small and collapsible, making it easier for pathogens to cause an infection.

At which age should infants achieve proper eye coordination? Infants should achieve proper eye coordination by 2 weeks to 1 month. Infants should achieve proper eye coordination by 1-2 months. Infants should achieve proper eye coordination by 2 to 4 months. Infants should achieve proper eye coordination by 4 to 6 months.

Infants should achieve proper eye coordination by 4 to 6 months. Infants under 3-4 months of age often lack eye coordination and alignment but should achieve proper coordination by 4 to 6 months. By 3 months infants should be able to track and reach for objects as their eye-hand coordination begins to develop.

A nurse is caring for an adolescent patient who had Chlamydia during her pregnancy. The nurse will monitor closely for which postpartum complication? Infection Uterine myomas Chronic hypertension Postpartum hemorrhage

Infection

The nurse is preparing a patient for cesarean delivery. After the epidural anesthetic is initiated, the patient reports anxiety about being alone and awake in the operating room (OR) during delivery. Which nursing interventions are appropriate for this patient? Select all that apply. Reassure the patient that the procedure will not take very long. Inform the patient that a nurse will be present in the OR during the procedure. Describe the OR and the people who will be present during the delivery to the patient. Explain to the patient that because of the anesthesia, it is unlikely that the patient will remember very much about the delivery. Inform the patient that a family member is allowed in the OR for support, although he or she may have to wait a few minutes to come in.

Inform the patient that a nurse will be present in the OR during the procedure. The nurse preparing the patient is often the circulating nurse during cesarean delivery. Having a familiar face and voice present can reassure the patient. Describe the OR and the people who will be present during the delivery to the patient. Describing the OR and the people who will be present during delivery helps the patient know what to expect and may help alleviate anxiety. Inform the patient that a family member is allowed in the OR for support, although he or she may have to wait a few minutes to come in. The patient may have a support person in the OR during the delivery. Preparatory procedures such as catheter insertion may occur before the support person enters, which can take 30 minutes to 45 minutes. Knowing that a family member will be present may help alleviate the patient's anxiety.

A patient in prolonged labor expresses frustration when changing positions and states, "I didn't have to move around so much last time I had a baby." Which nursing intervention is most appropriate for this patient? Reduce the frequency of position changes to promote patient comfort Inform the patient that repositioning is necessary to promote progress of labor Educate the patient about how repositioning provides distraction, reducing pain perception Explain to the patient that repositioning is necessary to ensure adequate fetal oxygenation during prolonged labor

Inform the patient that repositioning is necessary to promote progress of labor Repositioning during prolonged labor is indicated to promote natural progress of labor.

Nursing Interventions During Third Stage of Labor

Instruct client to push once signs of placental separation are indicated Promote baby-friendly activities between family an neonate which facilitates release of endogenous maternal oxytocin Administer analgesics as prescribed Administer oxytocics once placenta is expulsed to stimulate uterine contractions and prevent hemorrhage Gently cleanse perineal area with warm water or 0.9% sodium chloride, apply perineal pad or ice pack

A new mother reports enlarged, hardened breasts. Which actions do the nurse take? Select all that apply. Instructs the mother to continue breastfeeding. Suggests the mother discontinue using breast pads. Encourages the mother to breastfeed every 2-3 hours. Tells the mother to avoid breastfeeding for at least 1 week. Encourages the mother to seek antibiotics from her health care provider.

Instructs the mother to continue breastfeeding. Breast enlargement and hardening are signs of engorgement. The mother should breastfeed regularly to ensure breast emptying. Encourages the mother to breastfeed every 2-3 hours. Breast enlargement and hardening are signs of engorgement. The mother should be encouraged to breastfeed more frequently to promote breast emptying.

A client exhibits oligohydramnios at 36 weeks' gestation. What newborn complication should the nurse anticipate? Spina bifida Imperforate anus Tracheoesophageal fistula Intrauterine growth restriction (IUGR)

Intrauterine growth restriction (IUGR) (Oligohydramnios is associated with IUGR; risk factors for IUGR include inadequate maternal nutrition and other high-risk conditions such as diabetes and preeclampsia. Spina bifida does not affect amniotic fluid volume; it is associated with an increased alpha-fetoprotein level. Imperforate anus does not affect amniotic fluid volume. Tracheoesophageal fistula is often associated with polyhydramnios, which is excessive amniotic fluid.)

An infant is born to a mother who abused heroin. Which neurological sign would the nurse assess for? Irritability Bradypnea Tachycardia Increased appetite

Irritability

The nurse is caring for the newborn of a mother with diabetes. For which signs of hypoglycemia should the nurse assess the newborn? Select all that apply. Pallor Irritability Hypotonia Ineffective sucking Excessive birth weight

Irritability Hypotonia Ineffective sucking (An inadequate amount of cerebral glucose causes irritability and restlessness. Hypoglycemia affects the central and peripheral nervous systems, resulting in hypotonia. Feeding difficulties result from hypoglycemic effects on the fetal central nervous system. Hypoglycemia causes cyanosis, not pallor, in the newborn. Excessive birthweight is common but does not indicate hypoglycemia.)

Match the method of heat loss with the nursing action that would prevent heat loss. -Keep newborn dry and covered -Place a blanket between the newborn and the exam table -Move the crib away from air vents -Move the crib away from the window or outside wall

Keep newborn dry and covered Evaporation Place a blanket between the newborn and the exam table Conduction Move the crib away from air vents Convection Move the crib away from the window or outside wall Radiation

A woman in labor appears to have used cocaine prior to arrival to the Labor and Delivery unit. The woman is agitated and wants to pace the halls. Which are the appropriate nursing interventions for this patient? Select all that apply. Avoid use of pain control medications. Keep the bed in a low, locked position. Pad the side rails and keep them up at all times. Explain that precautions are for the mother's own good. Reduce environmental stimuli (lights, noise) as much as possible.

Keep the bed in a low, locked position. Pad the side rails and keep them up at all times. Reduce environmental stimuli (lights, noise) as much as possible.

Which factors contribute to heat loss in the newborn? Select all that apply. Large skin surface Little subcutaneous fat Choice of feeding method Maternal temperature at birth Blood vessels close to the surface

Large skin surface Newborns have a large skin surface in proportion to their size, increasing the risk for heat loss. Little subcutaneous fat Newborns have inadequate stores of brown fat and are at a higher risk for heat loss due to lack of subcutaneous insulation. Blood vessels close to the surface Because of the lack of subcutaneous tissue, newborn blood vessels are closer to the surface, leading to increased insensible heat loss

When performing a newborn assessment, which finding should be reported to the health care provider? Vaginal bleeding Nipple discharge Large, protruding tongue 2 arteries and 1 vein in the umbilical cord

Large, protruding tongue A large, protruding tongue can indicate hypothyroidisim or a chromosomal abnormality and should be reported to the health care provider.

Nursing Assessments During First Stage

Leopold Maneuvers Vaginal exam as indicated to assess true labor and rupture of membranes Encourage client to take slow deep breaths prior to vag exam Monitor dilation and effacement Monitor station and fetal presenting part Prepare for impending delivery as the presenting part moves into + station and begins push against pelvic floor (crowning) Assessments related to Rupture of Membranes Assess FHR to ensure there is no fetal distress from possible umbilical cord prolapse which can occur with gush of amniotic fluid Verify presence of alkaline amniotic fluid using nitrazine paper (turns blue, pH 6.5-7.5) Obtain sample of fluid to view under microscope (amniotic fluid= frondlike ferning pattern), assess color and odor Perform Bladder Palpation Perform on regular basis to prevent distention which can impede fetal descent and can cause trauma to bladder Clients may not feel urge to void due to labor or anesthesia Encourage to void every 2 hrs Vitals, contractions and FHR monitoring Latent Phase: every 30 minutes Active Phase: Every 30 minutes Transition Phase: Every 15 minutes Temperature is taken every 4 hrs unless membranes are ruptured then they are taken every 1-2 hrs

An infant of a mother who abused cocaine during pregnancy may exhibit which symptoms? Select all that apply. Lethargy Agitation Inconsolability A resting heart rate of 165 beats per minute A systolic blood pressure greater than 113 mm Hg

Lethargy Agitation Inconsolability

The nurse is assessing a patient in the post-anesthesia care unit (PACU) following a cesarean delivery under general anesthesia. After confirming respiratory status, which is the next assessment the nurse should complete? Fundal height Color of lochia Blood pressure (BP) Level of consciousness (LOC)

Level of consciousness (LOC) Following confirmation of an adequate airway and respiratory status, LOC is next assessed. The patient needs to be alert and oriented in order to continue respiratory (breathing) effort. Consider the ABC principle of prioritization.

Which assessments should be the nurse complete for a neonate who has abdominal distension, bilious emesis, and bloody stools? Select all that apply. Assess abdominal girth daily. Feed the neonate 15 mL of formula. Listen to bowel sounds in all four quadrants. Assess hemoccult results on the current stool. Palpate abdomen for distended loops of bowel. Assess for grunting, tachypnea, and substernal retractions.

Listen to bowel sounds in all four quadrants. Assess hemoccult results on the current stool. Palpate abdomen for distended loops of bowel. Assess for grunting, tachypnea, and substernal retractions.

The mother of a newborn abused marijuana during her pregnancy. Which abnormal vital sign would be a common maternal finding? Elevated heart rate Increased blood pressure Increased respiratory rate Low oxygen saturation (Sa02)

Low oxygen saturation (Sa02)

A new mother is hesitant to breastfeed because she has inverted nipples. Which recommendations should the nurse make to promote latching? Select all that apply. Massage nipples prior to feeding Use a nipple shield during breastfeeding Increase the amount of water in your diet Allow newborn to chew the nipple prior to feeding Use a breast pump for a few minutes before feeding

Massage nipples prior to feeding Massaging nipples prior to feedings helps to evert the nipples and promotes latching. Use a nipple shield during breastfeeding Nipple shields are sometimes recommended for mothers with inverted nipples to promote latching Use a breast pump for a few minutes before feeding Use of a breast pump prior to feedings helps to evert the nipple for feeding and promotes latching.

Nursing Assessments During Fourth Stage of Labor

Maternal vital signs Fundus Lochia Urinary output Baby-Friendly activities of the family

Culturally Competent Care: Asian American

May prefer mother to be present; partner is not an active participant; labor in silence; C-section undesirable

Which examples are characteristics of Braxton Hicks contractions? Select all that apply. Increase with walking Menstrual-like cramping Consistently increase in frequency Are regular and last 40-60 seconds Occur every 5 minutes to 25 minutes

Menstrual-like cramping Braxton Hicks contractions typically increase with walking. Occur every 5 minutes to 25 minutes Braxton Hicks contractions are typically irregular and infrequent.

A woman who is at 36 weeks' gestation thinks she is experiencing labor. Which signs or symptoms would support the woman's suspicion? Select all that apply. Headache Menstrual-like cramps Reports of constipation Reports of pelvic pressure Reports that the baby is "balling up"

Menstrual-like cramps Reports of pelvic pressure Reports that the baby is "balling up

Because newborns are unable to shiver, they use which body process for warming? Vasodilation Decreased physical activity Metabolism of brown fat Increased respiratory rate

Metabolism of brown fat Nonshivering thermogenesis increases the newborn's body temperature by metabolizing brown fat

At 40 weeks' gestation, a woman calls, excited that she is "finally in labor." Which signs and symptoms support that she is not in labor? Select all that apply. Mild discomfort felt in the abdomen and pelvis Contractions that subside with increased activity No significant change in cervical effacement or dilatation A consistent pattern of contractions that increase in frequency and duration Discomfort that begins in the lower back and gradually sweeps around to the lower abdomen like a girdle

Mild discomfort felt in the abdomen and pelvis Discomfort during true labor typically begins in the lower back and gradually sweeps around to the lower abdomen like a girdle. Correct Contractions that subside with increased activity Walking tends to increase contractions during true labor. Correct No significant change in cervical effacement or dilatation The best distinction between true and false labor is that the contractions of true labor cause progressive changes in the cervix. Effacement and dilatation occur with true labor contractions.

Which head assessment finding is represented in the picture? Molding Bulging fontanels Cephalohematoma Separation of sutures

Molding This image shows molding, the shaping of the fetal head by overlapping of the cranial bones to facilitate movement through the birth canal during labor. This is a normal finding in newborns born during a vaginal delivery.

The nurse is preparing a patient for cesarean delivery because of umbilical cord prolapse. Which is the nurse's most appropriate action? Provide support to the patient Administer tocolytics as prescribed Monitor fetal heart rate (FHR) continuously Document the status of the fetus and mother at least every five minutes

Monitor fetal heart rate (FHR) continuously Cord prolapse presents an acute risk for fetal hypoxia, a problem that would manifest with decelerations in FHR. Monitoring FHR is a nursing assessment that directly addresses the most acute risk to the fetus and consequently should be prioritized.

A newborn is admitted to the nursery and classified as small for gestational age (SGA). What is the priority nursing intervention for this infant? Testing the infant's stools for occult blood Monitoring the infant's blood glucose level Placing the infant in the Trendelenburg position Comparing the infant's head circumference and chest circumference

Monitoring the infant's blood glucose level (SGA infants are prone to hypoglycemia, because they have little subcutaneous fat or glycogen stores. Intestinal bleeding is not common in SGA infants. Placing an SGA infant in the Trendelenburg position is of no therapeutic value. Hydrocephalus or microcephaly is not a characteristic of SGA infants.)

When assessing a newborn's respiratory system, the nurse should report which finding to the health care provider? Nasal flaring and grunting Cyanosis of the lower extremities Moist lung sounds on auscultation Respiratory rate of 42 breaths per minute

Nasal flaring and grunting Nasal flaring and grunting are abnormal assessment findings and should be reported to the health care provider. They may indicate impending respiratory distress syndrome or failure

A nurse is caring for an adolescent pregnant patient who is concerned about gaining weight with pregnancy. Which education should the nurse review? Calorie reduction Diets for during pregnancy Exercise during pregnancy Normal pattern of weight gain

Normal pattern of weight gain

A patient who has recently used heroin is admitted to the Labor and Delivery unit and is agitated, sweating profusely, and vomiting; which are priorities in nursing care? Refer her to a substance abuse clinic. Administer butorphanol immediately. Call nutrition services for a regular meal. Obtain an order to administer methadone.

Obtain an order to administer methadone.

The nurse is providing care for a patient in labor, and the health care provider has just stated the patient's need for a forceps-assisted delivery. What should the nurse's preparation include? Obtaining a urinary catheter Establishing intravenous (IV) access Performing a head-to-toe assessment Educating the patient about the risk for lacerations

Obtaining a urinary catheter The patient's bladder must be empty during an operative delivery. Use of an intermittent catheter is expected.

Five minutes after birth, a newborn is given an Apgar score of 8. Twelve hours later the newborn becomes hyperactive and jittery, sneezes frequently, and has difficulty swallowing. What does the nurse suspect is the cause of these clinical findings? Cerebral palsy Neonatal syphilis Opioid drug withdrawal Fetal alcohol syndrome

Opioid drug withdrawal (These adaptations indicate opioid drug withdrawal; the infant should be monitored for further withdrawal signs during the first 24 hours after birth. Signs of cerebral palsy usually manifest later in infancy. A low-grade fever and copious serosanguineous discharge from the nose are signs of syphilis. Growth deficiencies in length, weight, and head circumference are associated with fetal alcohol syndrome, as are certain facial abnormalities.)

The nurse is planning to use a newborn's foot to obtain blood for the required newborn metabolic testing. Which part of the foot is the best site to use for the puncture? Big toe Foot pad Inner sole Outer heel

Outer heel (The outer heel is well perfused and heals quickly. The big toe, foot pad, and inner sole are all inappropriate sites from which to obtain a blood specimen from a newborn.)

Match the woman to the relevant high-risk condition.

PTL - A woman at 28 weeks who is having regular uterine contractions. PROM - A woman at 38 weeks whose amniotic membranes have ruptured, but she has not experienced any contractions. PPROM - A woman at 32 weeks whose amniotic membranes have ruptured. Chorioamnionitis - A woman at 33 weeks who has a temperature of 101.5 degrees F and abdominal tenderness.

The nurse touches the base of the newborn's fingers and notes the fingers flex and close into a tight fist. Which reflex is being elicited? Gallant reflex Rooting reflex Tonic neck reflex Palmar grasp reflex

Palmar grasp reflex The palmar grasp reflex is assessed by touching the base of the fingers. The fingers should flex into a tight fist.

While performing a vaginal examination on a patient in active labor, which finding would prompt the nurse to call the health care provider immediately? Soft cervical consistency Palpable fetal head behind the dilated cervix Increased cervical dilation since previous examination Palpable pulse that beats synchronously with fetal heart monitor

Palpable pulse that beats synchronously with fetal heart monitor A palpable pulse that beats synchronously with the fetal heartbeat indicates a prolapsed umbilical cord; the nurse is feeling the umbilical cord during this examination, which indicates a prolapse.

A newborn is admitted to the nursery. The newborn weighs 10 lb, 2 oz (4592 g), which is 2 lb (907 g) more than the birthweight of any of the neonate's siblings. Which intervention should the nurse implement in relation to this baby's birth weight? Document the findings Delay starting oral feedings Perform serial glucose readings Place the newborn in a heated crib

Perform serial glucose readings (A large newborn may be the result of gestational diabetes; it is necessary to check the neonate for hypoglycemia, because maternal glucose is no longer available. The nurse should do more than document the findings; the primary healthcare provider should be notified after the serial glucose readings are taken. Placing the infant in a heated crib is indicated if the temperature is low and the newborn needs additional warmth. The infant may be hypoglycemic and require the glucose in an oral feeding immediately.)

The nurse assesses the genitalia of a newborn female and should report which findings to the health care provider? Select all that apply. Perineal skin irritation Diminished urine output Small amount of vaginal discharge Genitalia darker than the surrounding skin Labia majora completely cover the clitoris and labia minora

Perineal skin irritation Perineal skin irritation is not expected in the newborn female and would be reported to the health care provider. Diminished urine output Diminished urine output is not expected in the newborn female and would be reported to the health care provider.

While waiting for a cesarean delivery because of cord prolapse, a patient in the Trendelenburg position reports discomfort and asks for her head to be lifted up. The nurse offers to reposition her onto her side with pillows supporting her hips. Which is the nursing rationale for this action? Pillows maintain elevation of the hips. A side-lying position relieves pressure on the patient's back. The head should not be lifted to prevent orthostatic hypotension. A side-lying position reduces the risk of precipitate birth while awaiting a cesarean section.

Pillows maintain elevation of the hips. Using pillows to elevate the hips in a side-lying position helps maintain fetal oxygenation by relieving pressure on the umbilical cord.

A neonate has an Apgar score of 2 at 1 minute of life and 6 at 5 minutes of life. The neonate requires oxygen and IV fluid therapy. Which is the priority intervention for this neonate? Obtain blood pressure and vital signs. Apply a continuous pulse oximeter to the left foot. Discuss the treatment plan with the parents when they visit the neonatal intensive care unit (NICU). Place the neonate under a radiant warmer with a temperature probe attached to the right upper quadrant of the abdomen.

Place the neonate under a radiant warmer with a temperature probe attached to the right upper quadrant of the abdomen.

The nurse notes a 2-hour-old newborn is grunting and lethargic with a respiratory rate of 72. Which actions does the nurse immediately implement? Select all that apply. Allow the mother to breastfeed Place the newborn in an upright position Administer albuterol nebulizer treatment Suction the oral cavity with a bulb syringe Apply a pulse oximeter and apnea monitor

Place the newborn in an upright position Newborns who are grunting should be repositioned to facilitate lung expansion. Suction the oral cavity with a bulb syringe Grunting newborns should have their mouths and noses suctioned to remove mucus and fluids retained from delivery. Apply a pulse oximeter and apnea monitor Grunting and tachypnea are signs of respiratory distress. The newborn should be monitored closely for signs of distress.

While taking an axillary temperature of a newborn, the nurse notes a temperature of 96.62 degrees Fahrenheit. Which actions do the nurse take? Select all that apply. Place the newborn under a warmer Obtain a rectal temperature and compare Allow for skin-to-skin contact with the mother Place more clothes on the newborn when sleeping Keep the legs covered when assessing the abdomen

Place the newborn under a warmer Placing the newborn under a warmer helps to increase the temperature quickly. Allow for skin-to-skin contact with the mother Skin-to-skin contact helps to increase the newborn's temperature and promotes mother-child bonding. Keep the legs covered when assessing the abdomen The nurse should avoid exposing more of the body than necessary during assessment.

A woman at 34 weeks' gestation presents with painful uterine contractions, lower back pain, and bloody vaginal discharge. Additionally, the woman has a temperature of 102° F and is noted to be at 2 cm dilated. Which action should the nurse take first? Assess for infection. Provide emotional support. Place the woman on bed rest. Evaluate fetus for distress, size, and maturity.

Place the woman on bed rest.

A mother of a 3-month-old infant is requesting information on the motor milestones she can expect to see in her child. The nurse can tell the parent to expect to see which activities at 3 months? Select all that apply. Puts foot in mouth Places hand in mouth Turns from back to abdomen Turns from abdomen to back Can lift head off bed in prone position.

Places hand in mouth The nurse can tell the parent that at 3 months the child will attempt to place their hand in the mouth. Can lift head off bed in prone position. By 3 months the infant should be able to lift the head off a bed in prone position, so the nurse can tell the parent to expect this activity.

Which patient laboratory value would the nurse validate as part of routine preparation for cesarean delivery, in order to determine adequate fetal maturity? Rh-positive antibody test performed 4 weeks prior to surgery. Negative rubella titer performed 6 weeks prior to surgery. Positive serum pregnancy test performed at least 36 weeks prior to surgery. High serum alpha-fetoprotein (AFP) level performed 26 weeks prior to surgery.

Positive serum pregnancy test performed at least 36 weeks prior to surgery. Positive serum pregnancy test, performed at least 36 weeks prior to the date of cesarean delivery, can be used to verify gestational age.

A 16-year-old pregnant patient gives birth to a 2045-gram baby. Which factors likely contributed to the birthweight of the baby? Select all that apply. Preeclampsia Preterm delivery Cigarette smoking Late prenatal care Low socioeconomic status

Preeclampsia Preterm delivery Cigarette smoking

Culturally Competent Care: African American

Prefer female family members for support

Culturally Competent Care: Native American

Prefer female nursing personnel; family involved in birth; use of herbs during labor; squatting position for birth

Culturally Competent Care: Hispanic

Prefer mother to be present during birth rather than partner

A nurse caring for an adolescent mother asks the patient if she has chosen a pediatrician for her newborn. This question addresses which aspect of the patient's cognitive development? Egocentrism Cause and effect Abstract thinking Present-future orientation

Present-future orientation

The nurse is testing newborns' heel blood for the level of glucose. Which newborn does the nurse anticipate will experience hypoglycemia? Select all that apply. Preterm infant Infant with Down syndrome Small-for-gestational-age infant Large-for-gestational-age infant Appropriate-for-gestational-age infant

Preterm infant Small-for-gestational-age infant Large-for-gestational-age infant (Preterm infants have low glycogen stores. Small-for-gestational-age infants also have low glycogen stores. Large-for-gestational-age infants are prone to hyperinsulinemia; often they are born to mothers who have diabetes, meaning that they are exposed to a high circulating glucose level while in utero. After prolonged exposure to a high glucose level, hyperplasia of the pancreas occurs, resulting in hyperinsulinemia. Infants with Down syndrome are not at risk for hypoglycemia but are at risk for congenital cardiac defects. Appropriate-for-gestational-age infants are not at risk for hypoglycemia.)

A woman at 34 weeks' gestation reports having four contractions in one hour and menstrual-like abdominal cramps. Which condition is the woman experiencing? Stillbirth Miscarriage Chorioamnionitis Preterm labor (PTL)

Preterm labor (PTL)

Nursing Interventions During First Stage of Labor

Provide teaching to clients about what to expect during labor Implement relaxation measures Encourage upright position, application of warm/cold packs, ambulation, or hydrotherapy if not contraindicated Encourage voiding at least every 2 hrs During Active Phase: Client/fetal monitoring Frequent position changes Encourage voiding Encourage deep cleansing breaths/ relaxation Provide nonpharmacological comfort measures Provide pharmalogical pain relief as prescribed During Transition Phase: Continue to encourage voiding Continue monitoring/ supporting client and fetus Encourage rapid pant-pant-blow breathing Discourage pushing efforts until cervix is fully dilated Listen for client statements of bowel movements which is a sign of fetal descent and prepare for birth Observe for crowning Encourage bearing down when cervix is dilated

The nurse is caring for a patient who had a forceps delivery that caused a perineal hematoma. Which nursing intervention is most appropriate? Administer topical analgesic ointment as prescribed Provide the patient with an ice pack and educate her about its use Encourage the patient to lie on her side as much as possible until the injury heals Educate the patient about the fact that the hematoma was caused by the introduction of forceps

Provide the patient with an ice pack and educate her about its use Ice packs can help relieve the pain of hematomas or lacerations after operative delivery by causing vasoconstriction and decreasing blood flow to the area, as well as decreasing edema. Patients should be guided toward use for the first 12 hours, followed by intermittent use.

Parent--infant attachment is the most important association for which type of development? Biological Physiological Psychosocial Sociocultural

Psychosocial Psychosocial development needs to be reinforced in the infant's life. This is one of the most important aspects in the development of parent-infant attachment.

How can the parent-infant attachment be strengthened immediately after birth? The mother needs to have time alone to ensure she does not overwhelm the child. A mother should cuddle with the baby but allow the child to cry alone if the baby is upset. The mother should give the infant some space to allow for natural biological development. Reciprocal interactions that include holding the child, touching the child, and speaking gently to the child.

Reciprocal interactions that include holding the child, touching the child, and speaking gently to the child. This is an example of how the parent-infant interaction can be strengthened after birth and therefore a lot of positive physical and emotional interactions between the parent and child should be encouraged.

Which reported labor contraction description indicates it is time for a pregnant woman to proceed to the hospital? Mild discomfort felt in the abdomen and pelvis Contractions that subside with increased activity Regular contractions, 5 minutes apart, for 1 hour Irregular contractions, 20 minutes apart, mild in discomfort

Regular contractions, 5 minutes apart, for 1 hour During true labor a consistent pattern of contractions with increasing frequency, duration, and intensity usually develops.

A nurse is caring for a laboring patient with hydramnios. After the patient's membranes rupture, the fetal heart rate is 100 beats/min for longer than 90 seconds. The nurse suspects cord compression. Which action should the nurse perform first? Perform a vaginal examination Assess the patient's blood pressure Reposition the patient on her hands and knees with the head lowered Coach the patient in deep breathing exercises to promote oxygenation

Reposition the patient on her hands and knees with the head lowered Repositioning the patient onto her hands and knees with the head lowered relieves the pressure on the umbilical cord and maintains fetal oxygenation. Fetal hypoxia can have rapid and severe effects.

The nurse is caring for a patient experiencing prolonged labor. The patient reports intense back pain that radiates down to the leg. Which is the appropriate nursing intervention for this patient? Elevate the head of the bed. Notify the health care provider. Administer the ordered pain medication. Reposition the patient onto her hands and knees.

Reposition the patient onto her hands and knees. Back pain that radiates to the leg is also called "back labor", and it is often a sign of a fetus in the occiput posterior (OP) position. Repositioning the patient onto her hands and knees helps facilitate fetal rotation.

The nurse is caring for a patient who has just been admitted to the labor and delivery unit. The patient suddenly grips the side rails and yells loudly, and the nurse can see the infant rapidly crowning. Which action should the nurse take first? Contact the health care provider. Reposition the patient to a side-lying position. Administer oxygen to the patient through a face mask. Reposition the patient so her hips are elevated above the head.

Reposition the patient to a side-lying position. A patient experiencing precipitate birth should remain in a side-lying position to slow the descent of the infant and prevent perineal lacerations. Repositioning the patient to a side-lying position is the first action the nurse should take because this is an independent nursing action that directly addresses the patient's priority health problem. This intervention can be performed rapidly, ensuring that other aspects of care are not delayed

A pregnant woman is admitted to labor and delivery unit after reports of preterm contractions. Which action is a priority for the nurse to take? Restrict the woman's activity Administer tocolytics immediately Provide patient education on preterm labor Call the radiology department for ultrasound

Restrict the woman's activity

A 30 weeks' gestation neonate is born to a mother who tested positive for group beta strep (GBS). The nurse would consider which interventions appropriate when caring for this neonate? Select all that apply. Obtaining vital signs once a shift Reviewing the complete blood cell count Weighing all weight diapers throughout the shift Obtaining a blood culture after starting antibiotics Instructing the parents to wash their hands before touching the newborn Administering ampicillin and gentamycin intravenously one after each other

Reviewing the complete blood cell count Weighing all weight diapers throughout the shift Instructing the parents to wash their hands before touching the newborn Administering ampicillin and gentamycin intravenously one after each other

Match the reflex being assessed with the picture.

Right upper picture: rooting Left upper picture: tonic neck Right lower picture: sucking Left lower picture: stepping

Which statement can the nurse use to educate the parent on leaving a child who shows emotional distress when having to leave the parent? Separations should be accomplished swiftly without dwelling on the separation. The mother should reciprocate and show the child that she too is emotionally a bit upset. Separations should not be done too fast as this can create emotional turmoil in the child. Separations should be accomplished swiftly, and with repeated emphasis on reassuring the child that the mother will return shortly.

Separations should be accomplished swiftly without dwelling on the separation. This statement suggesting the parent leave swiftly, yet with care, would help the parent-child interaction where the child is upset at leaving the mother.

The most appropriate method for a nurse to evaluate the effects of the maternal blood glucose level in the infant of a diabetic mother is by performing a heel stick blood test on the newborn. What specifically does this test determine? Blood acidity Glucose tolerance Serum glucose level Glycosylated hemoglobin level

Serum glucose level (Obtaining a blood glucose level is a simple, cost-effective method of testing newborns for suspected hypoglycemia. Although the acidity of the blood will indicate whether the newborn has metabolic acidosis as a result of hypoglycemia, it is more important to determine whether the newborn has hypoglycemia so it can be corrected before acidosis develops. The glucose tolerance test and glycosylated hemoglobin level test are not used in newborns.)

A 41-year-old pregnant patient lives alone, but reports a close-knit family consisting of elderly, retired parents in declining health and a 25-year-old sister who lives close by. Which concerns does the nurse have regarding the patient's support system? Select all that apply. She may not have peer support. She has no live-in support in her home. She may have limited help with childcare. Her mother may interfere with the patient developing her maternal role. Her parents may feel as though they have "failed" and may not be a source of support.

She may not have peer support. She has no live-in support in her home. She may have limited help with childcare.

When teaching the parents of a breastfed newborn about home treatment for jaundice, which information does the nurse include? Select all that apply. Signs of worsened jaundice Avoid exposure to the sun Breastfeed at least 8-12 times per day Allow the child to sleep through feedings Notify a health care provider if child does not void at least 6 times per day

Signs of worsened jaundice Parents should be taught the signs of worsening jaundice to report to the health care provider. Breastfeed at least 8-12 times per day Jaundice is the result of excess bilirubin in the blood. Frequent feeding promotes excretion of bilirubin through the GI tract. Notify a health care provider if child does not void at least 6 times per day Decreased urine output is a sign of worsening jaundice and should be reported to the health care provider.

The nurse is assessing the newborn of a known opioid user for signs of withdrawal. What clinical manifestations does the nurse expect to identify? Select all that apply. Sneezing Hyperactivity High-pitched cry Exaggerated Moro reflex Reduced deep tendon reflexes

Sneezing Hyperactivity High-pitched cry Exaggerated Moro reflex (Neurologic signs of withdrawal in the neonate of an opioid-addicted mother are manifested by sneezing, hyperactivity, jitteriness, and a shrill, high-pitched cry. The Moro reflex usually becomes exaggerated as the signs of withdrawal become apparent. The deep tendon reflexes are exaggerated during opioid withdrawal.)

A 41-year-old patient is seen for a preconception visit. Which education should the nurse review during this visit? Select all that apply. Sex education Stress management Vaccine recommendations Folic acid supplementation Cessation of tobacco and alcohol use

Stress management Vaccine recommendations Folic acid supplementation Cessation of tobacco and alcohol use

A nurse observes the newborn during feeding and is able to gather information about the newborn's ability to coordinate which actions? Select all that apply. Sucking Rooting Grasping Stepping Breathing

Sucking The nurse should observe the newborn's ability to suck during feedings and note an ability to coordinate feeding actions. Rooting The nurse should observe the newborn's ability to find the nipple at feeding time. Breathing The nurse should observe the newborn's ability to breathe during feedings and note an ability to coordinate feeding actions.

The nurse is caring for a newborn with a caput succedaneum. What is the priority nursing action? Supporting the parents Recording neurologic signs Applying a hard protective cap on the head Applying ice packs to the hematoma

Supporting the parents (Parents need support and reassurance that their newborn is not permanently damaged. Caput succedaneum does not cause impaired neurologic function. No special protection of the head is required; routine safety measures are adequate.)

A neonate experiencing neonatal abstinence syndrome (NAS) is irritable and has a high-pitched cry. Which intervention should the nurse utilize when caring for the neonate? Keep the neonate awake throughout the day shift. Allow the neonates arms to be free from blankets. Talk to the neonate frequently when providing nursing care. Swaddle the neonate snuggly in blankets with the arms flexed.

Swaddle the neonate snuggly in blankets with the arms flexed.

The nurse is assessing a newborn who had shoulder dystocia at delivery for a fractured clavicle. The nurse would expect which clinical signs if it is suspected the newborn has a fracture? Select all that apply. Swelling Warmth Redness Crepitus Flexed arm

Swelling Swelling or a lump over the clavicle is a potential sign of a fracture and indicates the need for further evaluation. Redness Redness over the clavicle is a potential sign of a fracture and warrants the need for further evaluation. Crepitus Crepitus, or a grating of the bone, is a sign of a clavicle fracture.

The nurse is caring for a patient who has just been prescribed intravenous (IV) oxytocin for the induction of labor. The nurse's subsequent assessments should address the risk for which complication of induction? Tachysystole Shoulder dystocia Fluid volume deficit Maternal hypertensive crisis

Tachysystole Oxytocin directly stimulates contractions, creating a risk for uterine hyperstimulation and tachysystole. For this reason, the nurse must prioritize assessments of the frequency, intensity, and duration of contractions.

The nurse is caring for a neonate whose mother is on methadone replacement therapy. Which interventions are appropriate for the nurse to utilize when caring for the neonate? Select all that apply. Talking to the newborn in a quiet voice Nipple feeding the neonate over 30 minutes Placing stimulating color patterns in the neonates crib Applying Vaseline to the buttock with each diaper change Completing the physical assessment and vital signs at the same time

Talking to the newborn in a quiet voice Applying Vaseline to the buttock with each diaper change Completing the physical assessment and vital signs at the same time

The nurse is caring for a new mother who presents with erythema over the right breast. The mother reports tenderness on palpation but denies history of fever. Which action does the nurse take? Apply cold compress Administer local anesthetic Teach breast massage technique Administer intramuscular antibiotic

Teach breast massage technique The patient's symptoms indicate a plugged mammary duct. The mother should be taught to massage the breast to relieve pain and unplug the milk duct.

The nurse is assessing a patient who had a cesarean delivery. The patient is silent during the physical assessment, but when asked how she feels responds, "I feel like I can't do anything right. How am I supposed to raise a child?" Which response from the nurse is most appropriate? "Tell me why you feel this way." "I can bring your baby in from the nursery so the two of you can bond." "Everyone is concerned about their first child. I can give you some printed material that can help." "I can give you contact information for a support group for new parents, if you feel overwhelmed."

Tell me why you feel this way." The nurse should open a dialogue to understand the patient's concerns. Often, patients can feel a sense of inadequacy following cesarean delivery due to failed expectations of having a normal vaginal birth.

Vital Signs

Temperature: -Axillary 36.5-37.3° C/97.7-99.1° F -Rectal 36.5-37.7° C/97.7-99.8° F -Skin 35.5-36.5° C/95.9-97.7° F Apical pulse: 120-160 beats per minute (bpm) while awake (100 sleeping, 180 crying) Respirations: 30-60 breaths per minute

The nurse notes that a breastfeeding mother stops feeding an newborn when he falls asleep after two to three minutes of suckling. Which information does the nurse include in patient teaching regarding feeding the newborn? Select all that apply. The baby needs to be awakened, so gently move the arms and legs of the newborn. This is a sign that the newborn is full, so breastfeeding should stop until the baby shows signs of hunger. Take the newborn off the breast and rub the newborn's back to help the baby get the amount of needed breast milk. The newborn is likely not interested in breastfeeding and the mother should switch to bottle feeding. Cessation of sucking while breastfeeding is a sign of nipple confusion, so the mother should provide pacifiers between feedings.

The baby needs to be awakened, so gently move the arms and legs of the newborn. Young newborns often fall asleep while breastfeeding. Moving the arms and legs can help to awaken the baby and help continuation of the feeding. Take the newborn off the breast and rub the newborn's back to help the baby get the amount of needed breast milk. The mother should be taught to wake the newborn and start again.

Place the feedback loop events responsible for labor contractions at term in the order of their expected occurrence.

The fetal head pushes against the cervix. The fetal head stretches the cervix. Oxytocin is increasingly secreted. The fundus of the uterus contracts.

Which physiological event is associated with lightening? The fetus assumes a position lower in the pelvis. The cervix begins to soften, dilate, and efface slightly. Fetal pressure causes congestion of the vaginal mucosa. The extra interstitial fluid that accumulates during pregnancy is excreted.

The fetus assumes a position lower in the pelvis. Lightening ("dropping") occurs as the fetus descends toward the pelvic inlet.

The nurse is performing a home visit for new parents. Which findings would indicate the need for further teaching and follow-up? Select all that apply. The mobile is placed 2 feet above the crib. The mother uses a football hold when breastfeeding. Older children assist with the baby's diaper changes. Parents are sitting watching television while child is awake and alert in a swing. The parents are having a dinner party with 60 people, while the newborn sits in a bouncy seat.

The mobile is placed 2 feet above the crib. Newborns focus best when things are 8-12 inches from their face. A mobile 2 feet above the crib will not engage the newborn. Parents are sitting watching television while child is awake and alert in a swing. The newborn needs social stimulation through verbal or physical interaction and should not be delegated to a swing when awake and alert. The parents are having a dinner party with 60 people, while the newborn sits in a bouncy seat. Too much stimulation, such as with a large party, can cause irritability.

An axillary temperature of 99 degrees Fahrenheit means what to the nurse? The newborn has a low temperature. The newborn has a high temperature. The newborn has a temperature within the normal range. The nurse should take a skin temperature instead to get a more accurate temperature.

The newborn has a temperature within the normal range. A temperature within the range of 97.7-99.1 degrees Fahrenheit is considered normal.

The nurse is caring for a patient who is 24 hours postpartum and whose delivery was aided by vacuum extraction. Which assessment finding should be reported to the health care provider? The patient has dark brown blotches on one side of her face. The patient has saturated three perineal pads in the past 4 hours. The patient's fundus is midline and below the umbilicus. The nurse has noticed a trickle of bright red blood for the past 2 hours.

The nurse has noticed a trickle of bright red blood for the past 2 hours. A slow, steady trickle of bright red blood can indicate a perineal laceration. This patient's history of operative delivery increases the risk of laceration; therefore, this finding should be reported to the health care provider.

A pregnant patient is crying and visibly upset when the nurse enters her room. The patient tells the nurse that she was just told her baby has Trisomy 18 and would die before or immediately after delivery. The patient tells the nurse she just doesn't know what to do. Which consideration would guide the nurse in responding to the patient?

The nurse is in the unique position to assist the patient in dealing with the emotional impact of a genetic disorder. As part of the genetic counseling team, the nurse helps family members deal with the emotional impact of a birth defect or syndrome. Recognizing this is a unique position to assist the patient allows the nurse to purposefully support the patient during this difficult time.

A patient came for genetic evaluation and was asked to provide information about family members with birth defects or chromosomal abnormalities. Which consideration should the nurse take as priority?

The nurse should consider that pertinent physical and medical information about family members is used to identify patients who would benefit from genetic counseling. Information about family members with birth defects or chromosomal abnormalities allows for identification of those who would benefit from genetic counseling.

A 2-month-old pediatric patient has been admitted to the hospital for the third time in 1 month because of complications related to a known congenital heart defect. The parents appear tired, and the nurse overhears them arguing about finances and who has sick time left. Which action is appropriate for the nurse to take?

The nurse should contact social services to provide appropriate support services. Utilizing social services will put the parents in contact with the needed support services to help them financially and emotionally.

A patient, who is undergoing genetic testing and counseling, tells the nurse she feels hopeless because the process has been so long and discouraging. She further confides to the nurse that she feels alone in this process. Which action should the nurse consider as priority?

The nurse should discuss support services available in the community. Genetic counseling utilizes a multidisciplinary approach with patients. The nurse should identify support groups and grief counselors in the area to assist the patient with her feelings.

The intrapartum nurse is caring for a laboring patient whose fetus is diagnosed with Trisomy 13. The patient tells the nurse, "My baby isn't going to live for long after being born," and begins crying. Which action should the nurse consider doing next?

The nurse should sit with the patient, provide support, and actively listen. Sitting with the patient and listening actively while she is talking demonstrates availability and genuine support.

During the initial prenatal appointment, a newly pregnant woman reports that her brother was born with Down syndrome. The woman is 32 years old and has experienced two miscarriages prior to this pregnancy. The woman asks the nurse if she should be concerned. What will the nurse consider after talking with this patient?

The nurse will consider that analysis of the entire family is necessary and genetic counseling should be discussed. The patient history provides information that must be acted upon. With the history of Down syndrome and miscarriages, the patient needs to obtain genetic counseling. Without counseling, it is hard to answer the question posed by the patient.

A nurse is caring for a patient following cesarean delivery. According to the operative report, the patient had a low transverse uterine incision. Which interpretation of this data by the nurse is accurate? The patient had minimal blood loss during the surgery. The patient is no longer a candidate for subsequent vaginal delivery. The patient is at risk for a uterine tear that could extend down into the cervix. The patient is at risk for uterine incisional rupture during a subsequent birth.

The patient had minimal blood loss during the surgery. A low transverse uterine incision is preferred since one advantage is that there is typically little blood loss during the procedure. This is related to its small size and placement on the uterus.

A patient experienced a first-degree laceration during vaginal delivery. Which postpartum assessment finding should the nurse attribute to the patient's laceration? The patient reports an overwhelming urge to defecate. The patient rates her overall pain as a 10 on a 10-point scale. The patient's pad contains bright red blood, as well as darker lochia. The patient reports lightheadedness when assisted to a high Fowler's position.

The patient's pad contains bright red blood, as well as darker lochia. In contrast to darker lochia, the blood from a laceration is more likely to be bright red, since it originates directly adjacent to the pad.

A client has delivered her infant by cesarean birth. The nurse monitors the newborn's respiration closely, because infants born via the cesarean method are prone to atelectasis. Why does this occur? The ribcage is not compressed and released during birth. The sudden temperature change at birth causes aspiration. There is usually oxygen deprivation after a cesarean birth. There is no gravity during the birth to promote drainage from the lungs.

The ribcage is not compressed and released during birth. (The release following compression of the chest during a vaginal birth is the mechanism for expansion of the newborn's lungs; because this does not occur during a cesarean birth, lung expansion may be incomplete, and atelectasis may result. Temperature change is not implicated in aspiration. The infant is monitored closely to prevent oxygen deprivation. The newborn's head may be held lower than the chest to allow gravity to promote drainage from the lungs after a cesarean birth.)

Which factors stimulate the respiratory center and initiate respirations at birth? Select all that apply. Genetic Thermal Sensory Chemical Mechanical Environmental

Thermal The newborn's adaptation to environmental temperature change from uterine to extrauterine life stimulates respiration. Sensory Tactile and auditory sensory stimulation of the nervous system help promote respiration in the newborn. Chemical Changes in oxygen levels create a chemical change in the newborn that stimulate respiration. Mechanical Mechanical factors, such as closure of cardiac shunts and changes in pressure in the heart and lungs, force fluid out of the lungs and stimulate respiration.

The nurse expects a newborn's stools to have which appearance immediately after birth? Clear, jelly-like Yellow, seedy Green, watery Thick, greenish black

Thick, greenish black Thick, greenish black stools are consistent with meconium and is expected immediately after birth.

A new mother's laboratory results indicate the presence of cocaine and alcohol. Which craniofacial characteristics indicate to the nurse that the newborn has fetal alcohol syndrome (FAS)? Select all that apply. Thin upper lip Wide-open eyes Small upturned nose Larger-than-average head Smooth vertical ridge in the upper lip

Thin upper lip Small upturned nose Smooth vertical ridge in the upper lip (The abnormal facial characteristics associated with FAS include: a thin upper lip (vermilion), a small upturned nose, and a smooth vertical ridge (philtrum) in the upper lip, all of which are distinctive in these infants. Infants with FAS have small eyes with epicanthic folds, rather than wide-open eyes, as well as microcephaly (head circumference less than the tenth percentile), rather than a larger-than-average head.)

A mother brings her 5-month-old infant to the clinic and is concerned that the child does not seem to smile as much as her niece who is the same age. How will the nurse respond? This is normal as you can expect to see a smile at 6 months of age. All children start to smile at a different age, so be patient. There is no need to worry. This is a concern since the child should be showing a social smile by no later than 1 month of age. This is a concern since the child should be showing a social smile by no later than 3 months of age.

This is a concern since the child should be showing a social smile by no later than 3 months of age. Since the child should be showing a social smile by no later than 3 months, this is the correct response that the nurse should make to this parent.

The newborn's initial breath requires a larger negative pressure for which reason? To promote clotting and prevent bleeding. To promote closure of cardiovascular shunts. To ensure gastric emptying and promote bowel elimination. To force the remaining pulmonary fluid out of the alveoli and into interstitial spaces.

To force the remaining pulmonary fluid out of the alveoli and into interstitial spaces. Negative pressure changes in the lungs force fetal lung fluid out of the alveoli to allow air into the lungs.

The labor and delivery nurse is monitoring a laboring patient and notes the presence of variable decelerations on the fetal monitor tracing. Which finding should the nurse expect based on this observation? Pressure on the fetal head Normal fetal heart activity Umbilical cord compression Uteroplacental insufficiency

Umbilical cord compression Variable decelerations present on a fetal heart monitor suggest umbilical cord compression.

Based on the assessment of a full-term infant, the nurse suspects a cardiac anomaly. Which clinical manifestation does the nurse identify that indicates a cardiac anomaly? Projectile vomiting Irregular respiratory rhythm Hyperreflexia of the extremities Unequal peripheral blood pressures

Unequal peripheral blood pressures (A discrepancy in blood pressures from the arms to the legs indicates arterial stenosis caused by coarctation of the aorta. Projectile vomiting commonly results from pyloric stenosis; it is not of cardiac origin and does not occur immediately after birth. An irregular respiratory rhythm is common and expected in the healthy newborn. Hyperreflexia of the extremities may be indicative of a neurologic, not cardiac, problem.)

Leopold Maneuvers

Used to determine position of fetus and estimate term fetal weight

Phototherapy is prescribed for a preterm neonate with hyperbilirubinemia. Which nursing intervention is appropriate to reduce the potentially harmful side effects of the phototherapy? Covering the trunk to prevent hypothermia Using shields on the eyes to protect them from the light Massaging vitamin E oil into the skin to minimize drying Turning after each feeding to reduce exposure of each surface area

Using shields on the eyes to protect them from the light (The lights used for phototherapy can damage the infant's eyes, and eye shields are standard equipment. Maximal effectiveness is achieved when the infant's entire skin surface is exposed to the light. Vitamin E oil massage is contraindicated, because it can cause burns and result in an overdose of the vitamin. The infant should be turned every 2 hours regardless of feeding times so that all body surfaces are exposed to the light and no single body surface is overexposed.)

The antepartum nurse is caring for a new patient who seeks to attempt a vaginal delivery after a cesarean delivery. She indicates her Pfannenstiel incision scar and insists that she should be able to successfully have a vaginal birth. Which nursing education is appropriate for this patient? Pfannenstiel incisions are very likely to rupture during subsequent birth. Vaginal birth may be attempted with a Pfannenstiel incision; however, labor will most likely need to be induced. Vaginal delivery may be possible, but her previous type of uterine incision would need to be verified before attempting vaginal delivery. Because her last cesarean was performed with a Pfannenstiel incision, she should be able to successfully have a vaginal birth.

Vaginal delivery may be possible, but her previous type of uterine incision would need to be verified before attempting vaginal delivery. Skin incisions and uterine incisions do not always match, and the patient's uterine incision may contraindicate subsequent vaginal delivery. Therefore, this nursing education is appropriate for the patient.

What physiological effect of cigarette smoking in a pregnant woman may lead to hypoxia in the infant? Vasodilation Vasoconstriction Restriction of cell growth Blockage of neurotransmitters

Vasoconstriction

The nurse cares for a mechanically ventilated 27 weeks' gestation neonate. The neonate desaturates and becomes bradycardic when suctioned and handled. Which interventions are most important for the nurse to implement when caring for this neonate? Select all that apply. Schedule suctioning for once in an hour. Warm and humidify the oxygen being administered. Nest the neonate in the supine position in the isolette. Increase the FiO2 from 30% to 50% before suctioning the neonate. Apply suctioning for 5 seconds at a time when withdrawing the suction catheter.

Warm and humidify the oxygen being administered. Increase the FiO2 from 30% to 50% before suctioning the neonate. Apply suctioning for 5 seconds at a time when withdrawing the suction catheter.

The nurse is preparing to discharge a 3-day-old infant who weighed 7 lb (3175 g) at birth. Which finding should be reported immediately to the healthcare provider? Weight of 6 lb 4 oz (2835 g) Hemoglobin of 16.2 g/dL (162 mmol/L) Three wet diapers over the last 12 hours Total serum bilirubin of 10 mg/dL (171 µmol/L)

Weight of 6 lb 4 oz (2835 g) (A loss of 12 oz (340 g) since birth, or more than 10%, is higher than the acceptable figure of 5% to 6%. Hemoglobin of 16.2 g/dL (162 mmol/L), total serum bilirubin of 10 mg/dL (171 µmol/L), and three wet diapers over the last 12 hours are all normal and expected findings.)

A neonate born at 32 weeks' gestation and weighing 3 lb (1361 g) is admitted to the neonatal intensive care unit. When should the nurse take the neonate's mother to visit the infant? When the infant's condition has stabilized When the infant is out of immediate danger When the primary healthcare provider has provided written permission When the mother is well enough to be taken to the intensive care unit

When the mother is well enough to be taken to the intensive care unit (The mother should see her infant as soon as possible so that she may acknowledge the reality of the birth and begin bonding. A delay impedes maternal-infant bonding. A prescription is not needed, because this is an independent nursing action. The infant's condition is not a controlling factor in determining when the mother initially visits.)

The nurse is visiting with a new mother who has chosen to breastfeed. The mother reports pain with suckling and removal of the newborn from the breast and that her milk supply seems low. Which recommendation does the nurse make? "The newborn is likely sleepy. You should wait until the baby is fully awake." "Use a breast pump for a few minutes before feedings to increase nipple size." "You should wait until the mouth is open wide to insert the nipple into the newborn's mouth." "Apply ointment to the nipple. It has a sweet taste and will prompt the newborn to breastfeed."

You should wait until the mouth is open wide to insert the nipple into the newborn's mouth." The mother should be taught that if the mouth is not open wide before the nipple is inserted, it is more difficult for the newborn to get sufficient milk and can cause nipple pain

A nurse is caring for a client immediately following an amniotomy. Which of the following interventions are appropriate? Select all that apply. Select one or more: a. Assess fetal heart for rate and variable decelerations. Correct b. Observe for the presence of an odor in amniotic fluid. c. Prepare for an intrauterine pressure catheter (IUCP) insertion. d. Assess maternal intake and urinary output. e. Document any unusual color in the amniotic fluid. Correct

a. Assess fetal heart for rate and variable decelerations. Correct b. Observe for the presence of an odor in amniotic fluid. e. Document any unusual color in the amniotic fluid. Correct After an amniotomy everything shifts in the uterus and the cord may become compressed. The nurse should immediately assess the fetal heart following the procedure. The presence of a foul odor in maternal fluid can be an important sign of infection. Any color in particular any meconium stained fluid or blood can indicate fetal distress.

A nurse is teaching a new mother breastfeeding techniques. Which of the following teaching tips are appropriate to discuss with a new mother who is breastfeeding? Select all that apply. Select one or more: a. Avoid a specific length of time to breastfeed. b. Dark, firm stools are the norm. c. Two to three wet diapers per day are the norm. d. Avoid use of a pacifier to prevent nipple confusion. e. Burp the newborn between each breast.

a. Avoid a specific length of time to breastfeed. d. Avoid use of a pacifier to prevent nipple confusion. e. Burp the newborn between each breast. Avoid educating mothers regarding the duration of newborn feedings. Mothers should be instructed to evaluate when the newborn has completed the feeding, including slowing of newborn suckling, a softened breast, or sleeping. Tell the mother to avoid nipple confusion in the newborn by not offering supplemental formula, pacifier, or soothers. Supplementation can be provided using a small feeding or syringe feeding, if needed. Show the mother how to burp the newborn when she alternates breasts. The newborn should be burped either over the shoulder or in an upright position with his chin supported. The mother should gently pat the newborn on his back to elicit a burp.

What are characteristics of the fetus that are reviewed to determine the biophysical profile (BPP) during an ultrasound? Select all that apply. Select one or more: a. Fetal tone b. Reactive FHR c. Qualitative amniotic fluid volume d. Fetal tidal volume e. Fine body movement

a. Fetal tone b. Reactive FHR c. Qualitative amniotic fluid volume Fetal tone, relative FHR, fetal breathing movements, gross body movements, fetal tone and qualitative amniotic fluid volume are physical and physiological characteristics of the BPP.

A nurse is caring for a client diagnosed with hyperemesis gravidarum. Which of the following are expected findings for this client? Select all that apply Select one or more: a. Ketosis b. Persistent diarrhea c. Dehydration d. Increased blood pressure e. Weight loss

a. Ketosis c. Dehydration e. Weight loss Hyperemesis gravidarum is excessive nausea and vomiting (related to elevated hCG levels) that is prolonged past 12 weeks of gestation and results in a 5% weight loss from prepregnancy weight, electrolyte imbalance, acetonuria, and ketosis. Dehydration would lead to a decrease in blood pressure and increase in pulse.

A nurse is caring for a neonate who is 34 weeks gestation. The nurse correctly understands which of the following are consistent with prematurity? Select all that apply. Select one or more: a. Large amount of vernix present b. Abundant lanugo c. Mongolian spots on shoulders d. Prominent clitoris and labia minora e. Inner eye canthus level with pina

a. Large amount of vernix present b. Abundant lanugo d. Prominent clitoris and labia minora Large amounts of vernix are noted with prematurity. Abundant lanugo is noted in abundant amounts with a premature newborn. Prominent clitoris and labia minora are seen with prematurity.

A nurse is administering magnesium sulfate to a client diagnosed with preeclampsia. Which of the following signs and symptoms would indicate possible magnesium toxicity? Select all that apply. Select one or more: a. Prolonged PR interval b. Hypertension c. Hypotension d. Diminished tendon reflexes e. Hyperactive tendon reflexes

a. Prolonged PR interval c. Hypotension d. Diminished tendon reflexes Magnesium Sulfate reduces striated muscle contractions due to a depressant effect on the CNS. It blocks neuromuscular transmission. Toxic signs of Magnesium sulfate include diminished tendon reflexes, hypotension and prolonged PR intervals.

A nurse is educating a client on how to perform Kegel exercise therapy for urinary incontinence. Which of the following points should be included in teaching? Select all that apply. Select one or more: a. While sitting on the toilet, strain down to help identify pelvic muscles. b. Have a designated time and place for completing therapy. c. Complete exercises in only a sitting position. d. Improvement in incontinence may be seen after 6 weeks of exercise therapy. e. During exercises, tighten pelvic muscles for a count of 10 and then relax for a count of 10.

b. Have a designated time and place for completing therapy. d. Improvement in incontinence may be seen after 6 weeks of exercise therapy. e. During exercises, tighten pelvic muscles for a count of 10 and then relax for a count of 10. At first, it is helpful to have a designated time and place to do exercises because the client will need to concentrate to do them correctly. Although improvement may take several months, most clients notice a positive change after 6 weeks of exercises. The client should be educated to tighten pelvic muscles for a slow count of 10 and then relax for a slow count of 10. This exercise should be done 15 times while lying down, sitting up, and standing (a total of 45 exercises). The client should then repeat the exercises rapidly contracting and relaxing the pelvic muscles 10 times. This should take no longer than 10 to 12 minutes for all three positions, or 3 to 4 minutes for each set of 15 exercises.

A nurse is providing prenatal education to a group of pregnant women. The nurse is teaching clients when to contact their provider. Which of the following should be included? Select all that apply. Select one or more: a. Evening lower extremity edema. b. Severe continuous headaches c. Chloasma d. Dimming vision e. Epigastric pain

b. Severe continuous headaches d. Dimming vision e. Epigastric pain All are possible symptoms of pregnancy induced hypertension. A severe and continuous headache along with visual changes could indicate CNS irritability and possible onset of seizures. Epigastric pain could indicate impending HELLP syndrome.

A laboring client reports suddenly feeling something in her vagina. Upon assessment, the nurse identifies a prolapsed umbilical cord. Place the following interventions in the correct order that they should be performed for this client. A. Prepare the client for a cesarean birth. B. Administer oxygen at 8-10L via face mask. C. Notify primary care provider of the prolapsed cord. D. Reposition the client in either a knee-ches or Trendelenburg position. E. Using a sterile glove insert two fingers into the vagina to reduce pressure off the cord. Select one: a. A, D, B, E, C b. B, C, D, A, E c. C, D, E, B, A d. B, A, D, C, E

c. C, D, E, B, A C. Notifying the health care provider and staff is the first priority and facilitates readiness for further interventions. D. Next step will be to remove pressure from the cord by repositioning client. E. Inserting fingers into the vagina and applying finger pressure to the fetal presenting part reduces pressure on the umbilical cord and provides oxygenation to the fetus. B. Administration of supplemental oxygen will further improve fetal oxygenation. A Emergent care of the client and fetus is priority and if all other measures fail, the client should be prepared for a cesarean birth.

A nurse is caring for a newborn client who is experiencing severe hyperbilirubinemia. Which of the following are symptoms of kernicterus? Select all that apply. Select one or more: a. Temperature instability b. Low birth weight c. Lethargy d. Backward arching of the neck and trunk e. Hypotonic

c. Lethargy d. Backward arching of the neck and trunk e. Hypotonic Kernicterus (bilirubin encephalopathy) can result from untreated hyperbilirubinemia with bilirubin levels at or higher than 25 mg/dL. It is a neurological syndrome caused by bilirubin depositing in brain cells. Survivors may develop cerebral palsy, epilepsy, or mental retardation. They may have minor effects such as learning disorders or perceptual-motor disabilities. Symptoms can inlcude letheragy, hyoptonia, high-ptiched cry and tonic motions such as backwards arching of the next and trunk. Low birth weight and temperature instabilty are not symptoms associated with kernicterus.

A nurse is educating a client who is scheduled for a nonstress test (NST). Which of the following statements are correct? Select all that apply. Select one or more: a. The NST is not useful after 38 weeks gestation. b. The NST is a useful in calculating gestational age. c. The NST can easily be performed in an outpatient setting. d. The NST measures the relationship of the fetal heart rate to fetal movement. e. The NST is a primary method of antenatal fetal assessment.

c. The NST can easily be performed in an outpatient setting. d. The NST measures the relationship of the fetal heart rate to fetal movement. e. The NST is a primary method of antenatal fetal assessment. In most settings the NST has become an ideal screening test for fetal well being. The NST is a noninvasive test which is easily performed in outpatient settings. The basis of the NST is the principle that the normal fetus will produce characteristic HR patterns in relationship to fetal movement.

Match the method of neonatal heat loss with its description.

liquid on skin converting to water vapor Evaporation laying the newborn on a cool weighing scale Conduction placing the newborn near an air vent Convection placing the newborn near a window or an outside wall Radiation

During the first year of life, the infant's ______ grow and mature at a rapid rate, yet an infant's organ _______ remain very different from those of older children and adults. Weight _____ and __________ ________ during infancy allow the infant to have increased control of reflexes and increasingly coordinated movement. Sensory capabilities, neuromuscular control, perceptual skills, the quality and quantity of parental interaction, and environmental stimulation all affect cognitive development during infancy. Infants develop language first by _________ __ _________ of caregivers, then by realizing that certain sounds have special meaning, and eventually by using simple words to communicate. By the end of the first year, an infant's social, cognitive, and motor skills have developed significantly, and appropriate nursing care and teaching will ensure parents are prepared for this growth and development.

organs systems gain muscle growth listening to sounds


Conjuntos de estudio relacionados

CA1: Introduction to Liabilities (True or False)

View Set

The Formation of Soil (Quiz Questions)

View Set

Chapter 4 (The Income Statement, Comprehensive Income, and The Statement of Cash Flows)

View Set

Тестові екзаменаційні завдання з навчальної дисципліни «Політична економія»

View Set

HIT Implementation Support Specialist

View Set

Women's Health/Disorders and Childbearing Health Protection

View Set